exam 4 pop health

अब Quizwiz के साथ अपने होमवर्क और परीक्षाओं को एस करें!

1. A resident in a rural community has been diagnosed with asthma. Which of the following providers will most likely provide care to this client? a. Advanced nurse practitioner b. Allergist c. Pediatrician d. Pulmonologist

A

11. A Mexican immigrant mother tells the nurse she is concerned that people in the community will give her daughter mal de ojo. To which of the following folk illnesses is the nurse referring? a. Evil eye b. Fallen fontanel c. Soul loss d. Indigestion

A

An American takes a long-awaited vacation in sunny Mexico, spending days on the beach eating fresh raspberries from a nearby vendor and drinking bottled water. The tourist may be altering: a) agent-host-environment interaction b) circadian rhythms c) herd immunity d) resistance

A The balance among agent, host, and environment is often precarious and may be unintentionally disrupted. Changes in the characteristics of any of the factors may result in disease transmission.

A nurse requests to meet a newly referred family in their home. Which of the following best explains the rationale for this request? (Select all that apply.) a. The nurse can assess the family environment. b. The family will feel more comfortable. c. Families typically welcome others into their home. d. More family members can typically be involved.

A B D

HIV transmission can occur through: a. exposure to blood. b. insect bites. c. sharing of school supplies. d. toilets.

ANS: A HIV can be transmitted through exposure to blood. HIV is not transmitted by insect bites, sharing of school supplies, or toilets.

A nurse screens blood products, donor organs, and tissues for the hepatitis C infection. Which of the following best describes this nursing action? a. Primary prevention b. Secondary prevention c. Tertiary prevention d. Health promotion

ANS: A Primary prevention refers to those interventions aimed at preventing the occurrence of disease, injury, or disability. Screening blood products, organs, and tissues for infection protects the population from exposure to hepatitis C, which prevents them from contracting the disease. Secondary prevention includes screening for diseases to ensure their early identification, treatment, and follow-up with contact to prevent further spread. Tertiary prevention focuses on chronic care and rehabilitation. Health promotion focuses on the primary prevention activities to promote health and prevent disease.

A nurse notes that the community has an unusually high prevalence of sexually transmitted infections among teens. Which of the following best describes a secondary prevention action the nurse could take?a.Conducting a sexual behavior survey with the adolescents b.Establishing in-school education related to transmission of sexual infections c.Providing free condoms at schools and universities d.Providing follow-up educational programs for those diagnosed with an STI

ANS: A Secondary prevention would include screening for risky behavior. Education and distribution of condoms are both primary prevention measures, and follow-up education for those diagnosed and being treated is tertiary to prevent further problems

A nurse wants to promote improved health for obese children in the community. Which of the following best describes a tertiary prevention measure that the nurse would implement? a. Establish lifestyle improvement programs through local youth organizations. b. Evaluate the food intake of a group of children for a 48-hour period. c. Provide education programs to overweight expectant parents. d. Evaluate the body mass index of children at regularly scheduled well-child exams.

ANS: A Tertiary prevention includes activities aimed to reduce the complications of the disease process. Only lifestyle improvement programs are directed toward preventing problems in children who are already obese. Evaluating food intake and evaluating BMI are types of screening programs (secondary prevention). Providing education programs to overweight expectant parents does not involve children.

A nurse is in the termination phase of the nurse-family relationship. Which of the following strategies would the nurse most likely implement? a.Increasing sessions with the nurse b.Making referrals when appropriate c.Providing a formative evaluation of the relationship d.Refusing additional communication with the family

ANS: B It also includes decreasing contact with the nurse, extending invitations to the family for follow-up, and a summative evaluation meeting for formal closure

A nurse is making an appointment with a family for a nursing visit. Which of the following describes a potential barrier the nurse may encounter? a.The assessment cannot be done unless the extended family is present. b.It may be difficult to find a convenient time for all family members to be present. c.Nurses have limited time to do home visits. d.Families are often scattered over a large area, making access difficult

ANS: B It is important to encourage all family members to attend the meeting. However, it can be difficult to find a convenient time for all family members to attend. Many times late afternoonor evening appointments are necessary to accommodate the needs of the family

A nurse wants to establish a program to decrease the death rate among adolescents. Which of the following programs should be developed? a. Anti-alcohol program b. Anti-smoking program c. Careful driving program d. Safe sex program

ANS: C

A community health nurse visits a homeless shelter to provide directly observed therapy (DOT) to several clients who have been diagnosed with TB. Which of the following best describes the rationale for this nursing intervention? a. Homeless clients do not care about their health. b. Homeless clients do not have access to medications. c. Poor adherence can result in drug resistance. d. These medications are so powerful, clients must be observed for reactions.

ANS: C Poor adherence has led to antibiotic resistant strains. These clients may care about their health but may have difficulty adhering to the treatment regimen. Medications to treat TB are available to the homeless population. The concern with the antimicrobial treatment is with non-adherence, not with side effects.

If underlying causes were listed on autopsy reports, which of the following would be the most common cause of unnecessary death in the United States? a. Alcohol b. Guns c. Tobacco d. Unprotected sex

ANS: C Smoking has been identified as the most important preventable cause of morbidity and mortality in the United States.

During which phase of the home visit does the nurse document what was accomplished? a.Pre-visit phase b.In-home phase c.Termination phase d.Post-visit phase

ANS: D A major task of the post-visit phase is documenting the visit and services provided.

Which of the following factors has the largest impact on health disparities among all populations? a. Ethnicity b. Education level c. Lifestyle choices d. Poverty

ANS: D Poverty is a strong and underlying current factor that affects all special groups.

The first stage of collaboration is: a. trust building. b. consensus. c. collegiality. d. awareness.

ANS: D The first stage of collaboration is awareness, when one makes a conscious entry into a group process. This is followed by tentative exploration and mutual acknowledgement, trust building, collegiality, consensus, commitment, and collaboration. REF: p. 496 (Box 22-4)

13. A nurse is implementing a tertiary prevention strategy related to pesticide exposure. Which of the following activities would the nurse complete? A. Observe farmworkers for evidence of unsafe handling of pesticides. B. Provide teaching on how to handle pesticides to avoid or decrease exposure. C. Teach farmworkers how to recognize signs and symptoms of pesticide poisoning. D. Treat a client who has pesticide exposure to prevent complications.

ANS: D Treat a client who has pesticide exposure to prevent complications. Tertiary prevention involves actions taken when a person already has a condition so that complications and worsening of the condition are avoided. DIF: Cognitive Level: Analyze (Analysis) REF: p. 395

A district health nurse is assigned to two rural communities in the state. To achieve the best outcomes possible in reducing the health disparities for the large number of frail elderly clients in the two counties, the nurse should consider using what community-oriented nursing approach?

Case management

During an outbreak of hepatitis A, nurses are giving injections of hepatitis A immunoglobulin to selected susceptible persons. Which of the following best describes the type of immunity that will follow the administration of these injections? a) Active immunity b) Long-lasting immunity c) Natural immunity d) Passive immunity

D Passive immunity refers to immunization through the transfer of a specific antibody from an immunized individual to a nonimmunized individual, such as the transfer of antibody by administration of an antibody-containing preparation (immune globulin or antiserum). Passive immunity from immune globulin is almost immediate but short-lived. It often is induced as a stopgap measure until active immunity has had time to develop after vaccination.

When using the health measure of death rates for working adults, the nurse could expect to find the highest death rates in which areas?

Most rural and highly populated urban areas

A clinic has received funds to pay for clinic visits for farm residents who live in frontier or rural, nonmetropolitan statistical areas. Which of the following client(s) would qualify to receive this special funding?

Soybean grower and his wife who live in a 400-square-mile county with a population of 39,501

A community health nurse is prioritizing health-promotion activities with the local rural community. An understanding of the health status and health risks of the rural community would lead the nurse to give priority to:

motor vehicle and farm accident prevention project and prenatal care outreach program.

A rural health nurse who is planning programs to address the population's needs should recognize that, in general, rural populations:

perceive their overall health as less favorable.

4. A nurse is caring for a migrant farm worker who has been working in the agricultural industry for the past 10 years. When questioned about environmental hazards, the client reports regular exposure to pesticides. Which of the following disorders is the client most at risk to develop? a. Cancer b. Memory loss c. Skin rashes d. Headaches

A

6. In addition to those barriers faced by many residents in rural areas, what additional barrier to health care is a Hispanic migrant farmworker likely to encounter? a. Absence of culturally competent care b. Availability of specialists c. Distance of health care facilities from the place of residence d. High cost of health care

A

3. A migrant farmworker has been injured while working on the farm. Which of the following would provide assistance to care for this injury? a. Emergency department for immediate care b. Migrant Health Act clinics c. OSHA, because the man was injured by farm machinery d. Workers' Compensation, because injury was at work

A

A new mother is a full-time college student who lives with her parents, because the baby's father has been imprisoned related to theft and drug abuse. The infant's grandmother, although also employed, cares for the child while the young mother attends classes. Which of the following theoretical frameworks would be most helpful to the nurse when assessing this family's needs? a. Developmental b. Interactional c. Structure-function d. Systems

A

A nurse considers how the environment outside of the family influences the development of a child when planning care for a family. Which of the following theories is being used by the nurse? a. Bioecological systems theory b. Family systems approach c. Family developmental theory d. Interactionist theory

A

A nurse is completing a tertiary prevention activity in a predominantly poor community, where eating clay (pica) is a common practice. Which of the following actions would the nurse most likely take? a. Assist those who eat large amounts of clay to obtain food stamps after explaining that clay, although filling, does not provide necessary nutrients. b. Initiate early intervention in the school system through education programs designed to focus on healthy food choices. c. Provide laboratory testing and physical assessments to assess for nutritional deficits resulting from clay intake. d. Survey families in the community to determine whether they eat clay and how much clay they eat.

A

In applying the developmental theory, a family nurse determines the developmental stage of the family based on: A. age of the eldest child. B. family strengths. C. individual growth patterns. D. overall tasks of the family.

A

The family nurse conducts the family nursing assessment with the family as a unit. Using a systematic process, family problems are identified and family strengths are emphasized as building blocks for interventions. Which of the following best completes the statement to demonstrate the importance of assessment to outcomes? Integrating the extended families: A. fosters equal family and provider commitment to success. B. facilitates outcomes-oriented family nursing research. C. decreases the need for nurse contact and intervention. D. removes barriers to needed services to achieve success.

A

The nurse in community health uses information about family structure, household composition, marriage, divorce, birth, death, adoption, and other family life events to forecast and predict stresses and developmental changes experienced by families and identify possible solutions to family challenges. This best describes the study of: A. family demographics. B. family functions. C. family health. D. family resilience.

A

Which of the following statements best explains why family functions and structures create unique challenges in family nursing? a. Function and structure change over time. b. Function and structure do not apply to all family units. c. Some clients do not have families. d. Traditional families are rare in society.

A

A nurse's Mantoux test is positive for exposure to tuberculosis. Which of the following conclusions should be drawn by the nurse? a) The nurse has been exposed to tuberculosis b) The nurse has tuberculosis c) The positive test result probably is due to a problem in the testing process d) The test is inaccurate and needs to be repeated

A An individual who tests positive has been exposed and may be infected, but if that person shows no clinical signs, the person is not diseased. Infection refers to the entry, development, and multiplication of the infectious agent in the susceptible host. Disease is one of the possible outcomes of infection. People with latent TB have no symptoms, are not infectious, and can continue on with life. They may develop active TB. A nurse who interacts with clients may receive INH for a year as a precaution.

Which of the following biological warfare agents poses the greatest bioterrorism threat to a community? a) Anthrax b) Botulism c) Smallpox d) Tularemia

A Because of factors such as the ability to become an aerosol, the resistance to environmental degradation, and a high fatality rate, inhalational anthrax is considered to have an extremely high potential for being the single greatest biological warfare threat.

Protecting the nation's food supply from contamination by all the virulent microbes is complex, costly, and time consuming. However, much foodborne illness, regardless of causal organisms, can be prevented through simple changes in: a) food preparation, handling, and storage b) importation regulations c) pesticide usage d) animal breeding practices

A Because safe food preparation measures are so important in preventing foodborne disease, the World Health Organization (WHO) has developed the "Ten Golden Rules for Safe Food Preparation." Many foodborne illnesses, regardless of causal organism, can be prevented by simple changes in food preparation, handling, and storage to destroy or denature contaminants and prevent their further spread. Because these measures are so important in preventing foodborne disease, Healthy People 2020 has continued to include an objective directed toward food safety.

Universal Precautions is a policy for all health care settings, where potential contact with blood or other body fluids exists, and requires that health care workers always perform hand hygiene and wear gloves, masks, protective clothing, and other indicated personal protective barriers. The underlying strategy for this policy requires that: a) blood and body fluids of all clients be handled as if infected b) health care workers effectively use hand hygiene c) health care settings are reservoirs of infection d) effective infection control surveillance programs are in place

A In 1985, in response to concerns regarding the transmission of HIV infection during health care procedures, the Centers for Disease Control and Prevention (CDC) recommended a Universal Precautions policy for all health care settings. This strategy requires that all blood and body fluids from all clients be handled as if infected with HIV or other blood-borne pathogens.

Which of the following is the number one cause of death worldwide? a) Chronic diseases (heart disease, cancer, stroke) b) Infectious diseases c) Injuries (accidental or purposeful) d) Terrorism

A In countries with higher standards of living, where people live longer, chronic diseases—heart disease, cancer, and stroke—are the leading causes of death. Infectious diseases, however, are still the number-one cause of death worldwide.

Society bears the burden of infectious disease. That burden includes the effects of morbidity and mortality as well as the staggering: a) economic burden b) fear burden c) incidence burden d) vulnerability burden

A Infectious diseases are expensive. The Produce Safety Project at Georgetown University in 2010 suggested that the total cost from foodborne illness could be more than $152 billion.

Which of the following is the most probable cause of the increase in new emerging infectious diseases? a) Activities or behavior of humans, including changes in the environment b) Increasing urbanization and growth in new housing materials c) New infectious agents are evolving throughout the world d) Overpopulation in many areas, creating a need to reduce global population

A Most of the emergence factors are consequences of activities and behavior of the human hosts and of environmental changes such as deforestation, urbanization, and industrialization. For example, the rise in households with two working parents has increased the number of children in daycare, and with this shift has come an increase in diarrheal diseases such as shigellosis. Urbanization is not a problem, but increasing development into formerly unaffected areas such as rainforests is.

A client is using a primary prevention strategy to prevent infectious disease. Which of the following actions is the client most likely taking? a) A client receives a tetanus booster every 10 years b) A client receives a tetanus booster after stepping on a nail c) A client receives tetanus immunoglobulin after stepping on a nail d) A client with tetanus is given antibiotics and is placed on seizure precautions

A Tetanus boosters given before exposure are a measure of primary prevention because exposure has not yet occurred. If given after exposure (i.e., the client may be infected but disease has not developed), they are considered secondary prevention (similar to the textbook examples of immunoglobulin and rabies immunizations given after exposure). Immunoglobulin would be given if the client had not been previously immunized; however, this again is after exposure, so it is secondary prevention. Because the client has the condition, treatment is aimed at prevention of further injury.

Factors that contribute to newly emerging or reemerging infectious disease can be related to microbial adaptation and changes made by the infectious agent. However, most of the emergence factors are related to environmental changes and: a) consequences of human activities/behaviors b) increase in the number of vectors c) industrialization and urbanization d) unpredictable variances in the climate

A The interaction between the agent, host, and environment is the platform for determining the contributing factors to emerging and reemerging infectious disease. In some cases, the agent may be the source of change in balance. But in most cases it will be the environment (i.e., weather/climate, deforestation, urbanization, industrialization) or the consequences of the human host's activities or behaviors (illegal drug use, changing sexual behavior, legal and illegal immigration, travel, increase in the number of children in day care, inappropriate use of antibiotics).

Which of the following places best describes where the incidence of Vancomycin-resistant Staphylococcus aureus (VRSA) and methicillin-resistant S. aureus (MRSA) is currently rising? a) Areas where people share dressing or bathing facilities b) Daycare centers and schools c) Long-term care facilities d) Senior citizen centers

A Vancomycin-resistant Staphylococcus aureus (VRSA) and methicillin-resistant S. aureus(MRSA) remain problems for people who acquire the bacteria in the hospital, but there is a growing incidence of community-acquired MRSA in places where people closely share facilities such as locker rooms, prisons, and other close bathing areas.

Which of the following best represents an example of infectious disease spreading via a vector? a) Being bitten by an infected mosquito b) Disease spreading from infected mother to infant via the placenta c) A group of partygoers hugging and shaking hands d) Two persons, one of whom is infected, sharing a glass of soda

A Vertical transmission is the passing of infection from parent to offspring via placenta. Horizontal transmission is the person-to-person spread of infection through (among other ways) contact. Common vehicle refers to transportation of the infectious agent from an infected host to a susceptible host via food, water, milk, or other substance. Vectors include mosquitoes, which can transmit the infectious agent by biting the host.

What terms are used to describe healthy families? Select all that apply. A. Families with strengths B. Dysfunctional C. Functional families D. Resilient families E. Resistant families

A C D

Which of the following must be firmly established before beginning a family assessment? (Select all that apply.) a. Why the data are needed b. How best to interview each individual in the family c. The most convenient time for you to visit the family d. The rationale or purpose of the visit

A C D

A multisystem approach to community disease control would include such interventions as: (Select all that apply.) a) community action programs b) control of vectors c) improved surveillance systems d) legislation e) provision of chemotherapy

A, B, C, D, E The multisystem approach to communicable disease control has the following goals: (1) improve host resistance to infectious agents and other environmental hazards, (2) improve the safety of the environment, (3) improve the public health system, and (4) facilitate social and political changes to ensure better health for all people. All of the interventions above are examples of the activities that would meet these goals.

4. A nurse's family has moved to a rural area to be near her spouse's aging parents. Which of the following would the nurse most likely notice about the family's new environment? (Select all that apply.) a. It is very difficult to obtain continuing education, except for online programs. b. Neighbors seem to expect the nurse to know everything about all health care and illness conditions. c. Neighbors ask questions about health concerns any time they see the nurse, regardless of where they are. d. People come to the health clinic before their situation becomes relatively serious.

ABC

3. A nurse tells her nursing supervisor that her family is moving from the urban area where they both live to a rural area to be near her spouse's parents, who are becoming less independent. Which of the following suggestions would the nursing supervisor provide to the nurse? (Select all that apply.) a. "Community members will probably hold you in higher regard and will look up to you." b. "Expect to have less autonomy in a small town than you have working as a nurse in a medical center." c. "You may feel like an isolated outsider, because the community may not immediately accept you." d. "You will have to be very sensitive about the differences in the rural lifestyle."

ACD

1. Which of the following best describes why health professionals would be particularly concerned about the health needs of residents in rural areas? (Select all that apply.) a. About 25% of all U.S. residents live in rural settings. b. People in rural areas are especially susceptible to acute illnesses rather than chronic diseases. c. Diagnoses in rural areas are usually for physical injuries, not mental health concerns. d. A high prevalence of poverty exists among rural families.

AD

A client explains to the nurse that it is just impossible for her and her husband to continue to have his mother in the home alone during the day while they work because the woman becomes confused and has fallen twice. Which of the following community resources should the nurse recommend? a. Adult day health b. Home health c. Long-term care d. Senior center

ANS: A Adult day health is for individuals whose mental and/or physical function requires additional health care and supervision. It serves as more of a medical model than the senior center, and individuals typically return home to their caregivers at night. Long-term care would take the client out of the home. Home health would leave the patient for periods of unsupervised time in which the patient could have problems and help would not be available.

A nurse explained to a new mother that because she had tested positive for the hepatitis B virus, her newborn son would need the hepatitis B vaccine immediately and then also an immune globulin injection. "Wait," said the new mother. "Why is my son getting two shots?" Which of the following statements would be the best response by the nurse? a."One injection protects your son, while the other encourages his body to build up immunity." b."One shot keeps your son from getting sick, while the other is a typical vaccine to prevent you from accidentally infecting him." c."Since you've already been infected with the virus, your son needs twice as much protection." d."The second shot is just to make sure the first one works."

ANS: A Because infected persons may not have any symptoms, all pregnant women should be tested for HBsAg. If the mother tests positive, her newborn needs hepatitis B immune globulin to provide passive immunity and thus prevent infection. In addition, the newborn is given the hepatitis B vaccine at birth, with two follow-up injections, to build active immunity to the infection

Which of the following is the best way to ensure good nutrition in infants? a. Breastfeeding only b. Feed them only brand-name baby foods c. Feed them only brand-name cereals d. Feed them only brand-name formulas

ANS: A Breastfeeding is the preferred method of infant feeding. Breast milk provides appropriate nutrients and antibodies for the infant. Breastfed infants have fewer illnesses and allergies. Breastfeeding is associated with a lower risk in developing childhood obesity.

What are the six "rights" of case management? a. Care, time, provider, setting, price, and outcomes b. Patient, medication, route, time, documentation, and evaluation c. Place, setting, patient, plan, outcomes, and documentation d. Disease process, time, place, beneficence, advocate, and care provider

ANS: A Care, time, provider, setting, price, and outcomes are used to judge the effectiveness of case management. REF: p. 483

Which of the following children is most at risk for being abused? a. A 1-year-old b. A 6-year-old c. A 9-year-old d. A teenager

ANS: A Children under the age of 4 years and children with special needs are at highest risk for abuse.

A nurse is working with incarcerated adults who are being released from prison. Which of the following nursing interventions would be most appropriate for the nurse to implement? a. Connect offenders with community-based mental health programs. b. Provide community supervision for mentally ill offenders. c. Advocate for increased prison time to decrease recidivism rates. d. Educate about available state resources.

ANS: A Connecting offenders with community-based mental health programs at the time of release from prison can decrease recidivism rates, because many incarcerated adults experience major psychiatric disorders. It would be impossible for the nurse to supervise all mentally ill offenders in the community. Increased prison time is not shown to decrease recidivism rates and education about available state resources is not the most important nursing intervention to provide for this population.

A nurse has just witnessed the signing of an agreement between two parents in which the parents pledge not to yell at each other in the presence of their children. Which of the following is being demonstrated through this action? a.Contracting b.Crisis intervention c.Empowerment d.Strategic planning

ANS: A Contracting is making an agreement between those involved in a shared effort by both nurse and family. The premise of contracting is family control. It is assumed that when the family has legitimate control, their ability to make healthful choices is increased.

A new mother is a full-time college student who lives with her parents, because the baby's father has been imprisoned related to theft and drug abuse. The infant's grandmother, although also employed, cares for the child while the young mother attends classes. Which of the following theoretical frameworks would be most helpful to the nurse when assessing this family's needs?a.Developmental b.Interactional c.Structure-function d.Systems

ANS: A Developmental theory explains and predicts the changes that occur to humans or groups over time. Achievement of family developmental tasks helps individual members accomplish their tasks. In this case the new mother has tasks, whereas her parents have temporarily interrupted their progress in response to their daughter's (and grandchild's) needs.

A patient with a long history of asthma with many hospital admissions is referred to a case manager for disease management. Which of the following best describes the purpose of this referral? a. Chronic and costly disease conditions that require long-term care interventions b. Patients who cannot handle their disease c. Those who seek to control use by providing clients with correct information d. Patients who will need an advanced practice nurse instead of physician for monitoring

ANS: A Disease management activities target chronic and costly disease conditions that require long-term care interventions. Demand management seeks to control use by providing clients with correct information. Patients who cannot handle their disease or who need an advanced practice nurse instead of a physician do not explain the referral to disease management. REF: pp. 478-479

A nurse is examining a child in the early stages of HIV infection. Which of the following would the nurse expect to find? a. Failure to thrive and developmental delays b. Kaposi's sarcoma and developmental delays c. Toxoplasmosis and oral candidiasis d. Fatigue and shortness of breath

ANS: A Early symptoms of pediatric HIV infection include failure to thrive and developmental delays. Kaposi's sarcoma and oral candidiasis are common opportunistic diseases later in the disease process. Fatigue is a symptom seen with hepatitis and TB.

A school nurse asks a class about the ways HIV can be transmitted. Which of the following comments by a student indicates a need for additional teaching? a."I wouldn't sit next to someone with HIV." b."Having unprotected sex with someone who is infected spreads HIV." c."Sharing needles when shooting up drugs spreads HIV." d."Transfusions of blood products that are contaminated can spread HIV."

ANS: A HIV can be transmitted through exposure to blood, semen, vaginal secretions, and breast milk and by sharing needles, syringes, and other equipment used to prepare injectable drugs. It can also be spread by perinatal transmission from mother to child through delivery or breastfeeding and by transfusions of contaminated blood. HIV is not transmitted through casual contact such as touching or hugging someone who has HIV infection. It is not transmitted by insects, coughing, sneezing, office equipment, or sitting next to or eating with someone who has HIV infection. Worldwide, the largest number of HIV infections result fromheterosexual transmission

Which of the following best describes the current research findings related to the use of hormone replacement therapy (HRT)? a. HRT does not prevent heart disease. b. Use of HRT is recommended to prevent osteoporosis. c. When used with complementary therapies, HRT is most effective. d. HRT is a contributing cause of breast cancer.

ANS: A HRT does not prevent heart disease. To prevent heart disease women should avoid smoking, reduce fat and cholesterol intake, limit salt and alcohol, maintain a healthy weight, and be physically active.

A nurse is appraising health risks. Which of the following questions would most likely be asked by the nurse? a."Does your 4 year-old have a booster seat in the car?" b."Have you noticed any physical problems as you go about your daily routine?" c."What concerns do you have today? d."Why did you decide to come in for a checkup?"

ANS: A Health risk appraisal refers to the process of assessing for the presence of specific factors in each of the categories that have been identified as being associated with an increased likelihood of an illness, such as cancer, or an unhealthy event, such as an automobile accident

Healthy Communities and Cities is based on which of the following premises? a. When people have the opportunity to work out their own locally defined health problems, they will find sustainable solutions to those problems. b. When the health of a community is improved, the focus will be on life expectancy rather than quality of life. c. When health professionals assume a leadership role, the health of the community will improve. d. When cities recruit enough health professionals to care for those needing medical care, the cities health will improve.

ANS: A Healthy Cities is based on the premise that when people have the opportunity to work out their own locally defined health problems, they will find sustainable solutions to those problems. Healthy Cities aims to add to quality of life. Healthy Cities focuses on collaboration by all members of the community. Healthy Cities uses a multifaceted approach to problem solving when trying to improve health. REF: p. 443

The Healthy Communities and Cities initiative supports the idea that: a. healthy cities and communities must be both environmentally and socially sustainable. b. healthy public policy is the responsibility of elected officials. c. physical environments cannot be changed, thus strategies must be developed in order to cope with them. d. the World Health Organization (WHO) is the primary source of information for cities.

ANS: A Healthy cities and communities must be both environmentally and socially sustainable through a strategy of primary care. Healthy public policy should involve collaboration at all levels. Supportive environments should be created. The WHO originally began this movement, but now sharing occurs among those participating in this initiative. REF: p. 442

Which type of hepatitis would likely be found where sanitation is inadequate? a. A b. B c. C d. D

ANS: A Hepatitis A would likely be found where sanitation is inadequate. Hepatitis B and C are spread through blood and body fluids. Hepatitis D can only exist in people who are already infected with Hepatitis B.

A nurse wants to establish a program to decrease the death rate among children. Which of the following health problems should be the target of this program? a. Accidents and injuries b. AIDS c. Childhood obesity d. Vaccine-preventable diseases

ANS: A Injuries are the number one cause of death for children (and young adults up to age 21 years) in the United States. Injuries and accidents are the most important causes of preventable disease, disability, and death among children. Most are preventable. Obesity, although a significant problem, is not a common cause of death in children.

A nurse is uncomfortable discussing such topics as sexual behavior and sexual orientation when counseling clients and avoids this topic with clients. Which of the following is the most likely outcome of this avoidance? a. Potential risks and risky behaviors will not be identified. b. Transmission of STDs will remain unchanged. c. Clients will develop a trusting relationship with health care providers. d. The nurse will be violating the laws in several states.

ANS: A It is important that nurses be able to discuss these topics to help prevent and control STDs. Without discussion of these topics, it is possible that clients will not be aware that they have an STD and may transmit it to others. Thus, the transmission of STDs may increase. A trusting relationship with a health care provider may or may not develop and is not dependent on whether these issues are discussed. There are not laws that mandate nurses to discuss sexual behavior and sexual orientation with clients.

A nurse would like to learn more about the overall health of a population. Which of the following indicators would the nurse most likely use? a. Life expectancy b. Mortality rate c. Morbidity rate d. Health status

ANS: A Life expectancy is a measure that is often used to gauge the overall health of a population.

A nurse who is using population management needs to be able to work with integrated care delivery systems. Which of the following describes the rationale for this competency? a. Management has shifted from inpatient care to primary care providers as points of entry. b. Emphasis is on episodic illness care for individuals rather than on population management. c. Care management services and programs do not provide access and accountability, as provided by case management services. d. Assessment of the needs of the population is no longer necessary.

ANS: A Management has shifted from inpatient care as a point of entry to primary care providers as points of entry. The other statements are false. REF: p. 477

A nurse is assisting an employer who has hired an individual who has been recently diagnosed with HIV. Which of the following interventions would be most appropriate for the nurse to implement? a. Educate about how to reduce the risk of breaching the employee's confidentiality b. Explain how to inform coworkers about avoiding HIV transmission c. Facilitate obtaining medical insurance coverage for the HIV-infected employee d. Describe the early signs and symptoms of HIV infection

ANS: A Nurses frequently work in the education role, and employers may need assistance in dealing with HIV-infected employees. Disclosing a worker's infection to other workers, terminating employment, and isolating an infected worker are examples of situations that have led to litigation between employees and employers. Thus, the priority will be to protect the employer from litigation.

A public health nurse (PHN) is reviewing Healthy People 2020 to determine where to prioritize programming for the county health department. Based on Healthy People 2020, which of the following areas would the nurse most likely plan to implement programming? a. Reduce the rate of HIV transmission among adults and adolescents b. Eliminate sexually transmitted diseases (STDs) from developed countries c. Reduce deaths from gonorrhea d. Increase awareness about HIV in lesbian females

ANS: A One of the Healthy People 2020 objectives is reducing the number of cases of HIV infection among adults and adolescents. Eliminating STDs from developed countries, reducing deaths from gonorrhea, and increasing awareness about HIV among lesbian females are not addressed by Healthy People 2020.

Pelvic inflammatory disease (PID) is a common complication of: a. gonorrhea. b. syphilis. c. chlamydia. d. herpes.

ANS: A PID is a common complication of gonorrhea. PID is not a common complication of syphilis, chlamydia, or herpes.

A nurse is completing a tertiary prevention activity in a predominantly poor community, where eating clay (pica) is a common practice. Which of the following actions would the nurse most likely take? a.Assist those who eat large amounts of clay to obtain food stamps after explaining that clay, although filling, does not provide necessary nutrients b.Initiate early intervention in the school system through education programs designed to focus on healthy food choices. c.Provide laboratory testing and physical assessments to assess for nutritional deficits resulting from clay intake. d.Survey families in the community to determine whether they eat clay and how much clay they eat

ANS: A Tertiary prevention is undertaken to prevent additional health problems when a problem has occurred. Early intervention in the school system is an example of primary prevention. Lab testing and surveying families are screening activities to determine whether a problem is present and to catch it in the early phases; such screening activities are representative of secondary prevention

A nurse counsels a client to have the enzyme-linked immunosorbent assay (EIA) test. Which of the following best describes the rationale for this test? a. To indicate the presence of the antibody to HIV b. To reveal whether or not the client has AIDS c. To isolate the HIV virus d. To confirm HIV after having a positive Western blot

ANS: A The EIA is used to indicate the presence of the antibody to HIV. To minimize false positive results, the Western blot is used as a confirmatory test to verify the results. The EIA does not isolate the virus, nor does it reveal whether the individual has symptomatic AIDS.

Toward whom is the TLC model targeted? a. Caregivers of older persons with health problems b. Community organizations that offer services for the elderly c. Elderly clients with health problems d. Nurses who care for older clients

ANS: A The TLC model is focused toward caregivers (primarily families) in an effort to relieve caregiver burden. Components are T = training in care techniques, safe medication use, recognition of abnormalities, available resources; L = leaving the care situation periodically to obtain respite and relaxation and maintain their normal living needs; and C = care for themselves (the caregiver) through adequate sleep, rest, exercise, nutrition, socialization, solitude, support, financial aid, and health management.

A nurse considers how the environment outside of the family influences the development of a child when planning care for a family. Which of the following theories is being used by the nurse? a.Bioecological systems theory b.Family systems approach c.Family developmental theory d.Interactionist theory

ANS: A The bioecological systems theory describes how environments and systems outside of the family influence the development of a child over time

A public health nurse (PHN) is working with the chronically ill and families with young children needing age-specific health maintenance. Which of the following models of case management is being used? a. Client-focused b. System-focused c. Social service d. Long-term care

ANS: A The concern in client-focused models is with the relationship between case manager and client to support continuity of care and to access providers of care. System focused models address the structure and processes of using the population-based tools of disease management and case management plans to offer care for client populations. The social service models provide services to clients to assist them in living independently in the community and in maintaining their health by eliminating or reducing the need for hospital admissions or long-term care. Long-term care is not a model of case management. REF: p. 488

A nurse is working with a family member to reduce his health risk. Which of the following recommendations would most likely be made by the nurse? a."Be sure to take a 30-minute walk each day." b."Call our office if you have any questions or concerns at all." c."Come back in 2 weeks for follow-up on your surgery." d."Continue to take the drug until it is gone, even if you're feeling better earlier."

ANS: A The factors that determine or influence whether disease or other unhealthy results occur are called health risks. The major categories of risk include inherited biological risk, social and physical environmental risk, and behavioral risk. All the other options are treatment oriented rather than risk avoidance. Exercising for 30 minutes a day reduces the risk for many diseases

Which of the following statements best explains why family functions and structures create unique challenges in family nursing? a.Function and structure change over time. b.Function and structure do not apply to all family units. c.Some clients do not have families. d.Traditional families are rare in society

ANS: A The functions that families serve evolve and change over time. Some become more important and others less so. Family structures also change over time. The great speed with which changes in family structure, values, and relationships are occurring makes working with families at the beginning of the twenty-first century exciting and challenging

A community health nurse is planning to implement an intervention to reduce the prevalence of sexually transmitted diseases in the community. Which of the following actions would most likely be taken by the nurse? a. Establish immunization clinics to prevent STDs. b. Educate people with HIV about the mode of transmission. c. Explain to women that HIV is transmitted to women usually by IV drug abuse. d. Develop a STD clinic to increase community access to services.

ANS: A The goal of Healthy People 2020 is to promote responsible sexual behaviors, strengthen community capacity, and increase access to quality services to prevent sexually transmitted diseases and their complications. Nursing activities should align with these goals.

The goal of the Healthy Communities and Cities initiative is to: a. promote health through community participation. b. reorganize the current health care system. c. maximize the cost-benefit ratio of health care. d. improve the quality of care in communities. The goal of the Healthy Communities and Cities initiative is to: a. promote health through community participation. b. reorganize the current health care system. c. maximize the cost-benefit ratio of health care. d. improve the quality of care in communities.

ANS: A The goal of the Healthy Communities and Cities initiative is the promotion of health through community engagement and collaboration. This leads to addressing the health and quality of life for all through a process that includes diverse citizen participation, mobilization of all sections of the community, and community ownership. Reorganization of the current health care system, maximizing the cost-benefit ratio of health care, and improving the quality of care in communities are not part of the Healthy Communities and Cities initiative. REF: p. 442

A nurse has been successful in creating improvement in a family's health. Which of the following characteristics is most likely displayed by the nurse? a.Skilled at recognizing and strengthening the family's competencies b.Skilled at obtaining referrals and resources for the family c.Skilled at communication and interpersonal relationships d.Skilled at assessing and naming the family's main problems

ANS: A The nurse's approach to the family should be positive and focused on competencies rather than on problems or deficits.

The process of moving conflicting parties toward an outcome is called: a. negotiation. b. conflict management. c. problem-purpose expansion method. d. brainstorming.

ANS: A The process of moving conflicting parties toward an outcome is called negotiation. Conflict management has the goal of mutual benefit with limited loss, which is directed toward getting all parties to work together. The problem-purpose-expansion method is a way to broaden limited thinking by restating the problem and expanding the problem statement so that different solutions can be generated. In brainstorming, as many alternatives as possible are generated without placing a value on them. REF: p. 493

A nurse performs a community assessment as part of the Community Health Promotion Model. Which of the following best describes the rationale for this action? a. To become more acquainted with the multiple factors that influence health status b. To survey the citizens in the community about their wants and needs c. To allow special interest groups a say in health policy d. To provide a narrow focus for the committee work

ANS: A The purpose of including a community assessment in the implementation of the Community Health Promotion Model is to become more acquainted with the multiple biological, behavioral, social, and physical factors that describe and influence health status of residence. The assessment provides the frame of reference for identifying the community's strengths, needs, and resources. The community assessment involves multiple factors, which is more than surveying citizens or asking special interest groups for their opinions. The community assessment has a broad focus, not a narrow focus. REF: p. 451

A home health nurse who is visiting a family for the first time asks, "Could we review your extended family and other persons or groups with whom you interact each week?" Which of the following provides the best rationale for the nurse asking this question? a.To assess the family's environment and social resources and risks b.To communicate with relevant others as needed c.To determine financial assets available to the family in case of serious need d.To understand the extended family relationships

ANS: A The question by the nurse indicates that she is trying to obtain an ecomap. Ecomaps can provide information about relationships that the family has with others (such as relatives and neighbors), the family's connections with other social units (such as church, school, work, clubs, and organizations), and the flow of energy, positive or negative, in the family. An ecomap represents the family's interactions with other groups and organizations. Environmental or social risk and resources can be assessed from an ecomap

A male client visits the clinic office complaining of a yellow, green discharge from his penis. Which of the following STDs has the client most likely contracted?a.Gonorrhea b.Syphilis c.Herpes simplex virus 2 d.Human papillomavirus

ANS: A The symptoms for gonorrhea in a male include a burning sensation when urinating, or a white,yellow-green discharge from the penis. Some men may get swollen or painful testicles. In men, gonorrhea can cause epididymitis, a painful condition of the testicles that if untreated can lead to infertility

A nurse has completed health risk appraisals with several different families. Which of the following families would be of most concern to the nurse? a.An older couple who has just retired and sold their house, who talk about their new condo in a retirement community. b.Newlyweds who have been saving their money, who want to discuss birth control and family planning in preparation for future pregnancies. c.Parents who come with their child for his pre-kindergarten physical exam and want to be sure all the child's immunizations are up to date. d.A woman who is very pleased with her new position at the hospital and wants to have her pre-employment exam and drug screen

ANS: A Transitions (movement from one stage or condition to another) are times of potential risk for families. Age-related or life-event risks often occur during transitions from one developmentalstage to another. Transitions present new situations and demands for families. Moving from the family home to a smaller condo represents a major change in lifestyle. None of the other options represent major transitions. If the event is normative, or anticipated, it is possible for families to prepare for the event and its consequences.

Which of the following statements best explains why HSV-2 infection is more challenging fora client than gonorrhea infection? a.HSV-2 is a viral infection that is both chronic and incurable. b.HSV-2 is extremely expensive to treat. c.HSV-2, like HIV, is almost impossible to diagnosis in the early stages. d.Once a person has been treated for HSV-2, the person is immune to further outbreaks

ANS: A Unlike gonorrhea, there is no cure for HSV-2 infection; it is considered a chronic disease

A nurse is implementing Wagner's Chronic Care Model. Which of the following actions would the nurse most likely take? a. Educate a community group about hypertension control. b. Create a budget for chronic disease management. c. Administer immunizations to community members. d. Conduct depression screenings in the community.

ANS: A Use of electronic health records, provider reminders for key evidence-based care components, interprofessional teams communicating regularly and community health classes to educate people with chronic diseases are various ways the CCM is being implemented. Creating a budget is not a way to use CCM. Administration of immunizations and conducting depression screenings do not address the management of the most common and costly chronic diseases: heart disease, diabetes, stroke, cancer, and arthritis.

22. A client is using a primary prevention strategy to prevent infectious disease. Which of the following actions is the client most likely taking? A. A client receives a tetanus booster every 10 years. B. A client receives a tetanus booster after stepping on a nail. C. A client receives tetanus immunoglobulin after stepping on a nail. D. A client with tetanus is given antibiotics and is placed on seizure precautions.

ANS: A A client receives a tetanus booster every 10 years. Tetanus boosters given before exposure are a measure of primary prevention because exposure has not yet occurred. If given after exposure (i.e., the client may be infected but disease has not developed), they are considered secondary prevention (similar to the textbook examples of immunoglobulin and rabies immunizations given after exposure). Immunoglobulin would be given if the client had not been previously immunized; however, this again is after exposure, so it is secondary prevention. Because the client has the condition, treatment is aimed at prevention of further injury. DIF: Cognitive Level: Analyze (Analysis) REF: p. 487

6. In addition to those barriers faced by many residents in rural areas, what additional barrier to health care is a Hispanic migrant farmworker likely to encounter? A. Absence of culturally competent care B. Availability of specialists C. Distance of health care facilities from the place of residence D. High cost of health care

ANS: A Absence of culturally competent care Barriers to health care in general affecting all populations include whether services and professionals are available, affordable, or accessible to rural consumers. For migrant workers, a language barrier and cultural differences often exist between them and other area residents, including health care providers. DIF: Cognitive Level: Understand (Comprehension) REF: p. 395

12. Which of the following is the most probable cause of the increase in new emerging infectious diseases? A. Activities or behavior of humans, including changes in the environment B. Increasing urbanization and growth in new housing materials C. New infectious agents are evolving throughout the world D. Overpopulation in many areas, creating a need to reduce global population

ANS: A Activities or behavior of humans, including changes in the environment Most of the emergence factors are consequences of activities and behavior of the human hosts and of environmental changes such as deforestation, urbanization, and industrialization. For example, the rise in households with two working parents has increased the number of children in daycare, and with this shift has come an increase in diarrheal diseases such as shigellosis. Urbanization is not a problem, but increasing development into formerly unaffected areas such as rainforests is. DIF: Cognitive Level: Understand (Comprehension) REF: pp. 484-485

1. A resident in a rural community has been diagnosed with asthma. Which of the following providers will most likely provide care to this client? A. Advanced nurse practitioner B. Allergist C. Pediatrician D. Pulmonologist

ANS: A Advanced nurse practitioner The providers most often seen by rural adults are general practitioners and advanced practice registered nurses (APRNs). In contrast, urban adults are more likely to seek care from a medical specialist. DIF: Cognitive Level: Apply (Application) REF: p. 393

14. Which of the following biological warfare agents poses the greatest bioterrorism threat to a community? A. Anthrax B. Botulism C. Smallpox D. Tularemia

ANS: A Anthrax Because of factors such as the ability to become an aerosol, the resistance to environmental degradation, and a high fatality rate, inhalational anthrax is considered to have an extremely high potential for being the single greatest biological warfare threat. DIF: Cognitive Level: Understand (Comprehension) REF: pp. 487-488

2. Which of the following places best describes where the incidence of Vancomycin-resistant Staphylococcus aureus (VRSA) and methicillin-resistant S. aureus (MRSA) is currently rising? A. Areas where people share dressing or bathing facilities B. Daycare centers and schools C. Long-term care facilities D. Senior citizen centers

ANS: A Areas where people share dressing or bathing facilities Vancomycin-resistant Staphylococcus aureus (VRSA) and methicillin-resistant S. aureus (MRSA) remain problems for people who acquire the bacteria in the hospital, but there is a growing incidence of community-acquired MRSA in places where people closely share facilities such as locker rooms, prisons, and other close bathing areas. DIF: Cognitive Level: Understand (Comprehension) REF: p. 478

6. Which of the following best represents an example of infectious disease spreading via a vector? A. Being bitten by an infected mosquito B. Disease spreading from infected mother to infant via the placenta C. A group of partygoers hugging and shaking hands D. Two persons, one of whom is infected, sharing a glass of soda

ANS: A Being bitten by an infected mosquito Vertical transmission is the passing of infection from parent to offspring via placenta. Horizontal transmission is the person-to-person spread of infection through (among other ways) contact. Common vehicle refers to transportation of the infectious agent from an infected host to a susceptible host via food, water, milk, or other substance. Vectors include mosquitoes, which can transmit the infectious agent by biting the host. DIF: Cognitive Level: Apply (Application) REF: p. 480

4. A nurse is caring for a migrant farm worker who has been working in the agricultural industry for the past 10 years. When questioned about environmental hazards, the client reports regular exposure to pesticides. Which of the following disorders is the client most at risk to develop? A. Cancer B. Memory loss C. Skin rashes D. Headaches

ANS: A Cancer Chronic exposure to pesticides and chemicals may lead to cancer. DIF: Cognitive Level: Apply (Application) REF: p. 393

3. A migrant farmworker has been injured while working on the farm. Which of the following would provide assistance to care for this injury? A. Emergency department for immediate care B. Migrant Health Act clinics C. OSHA, because the man was injured by farm machinery D. Workers' Compensation, because injury was at work

ANS: A Emergency department for immediate care Farming and ranching do not often fall under OSHA guidelines, because they are considered small enterprises. Therefore, safety standards are not enforceable, nor is Workers' Compensation insurance usually available for the agricultural industry. Although there are migrant health clinics, they are not always geographically convenient. The only source the nurse can count on using is sending injured workers to emergency departments for immediate care. DIF: Cognitive Level: Apply (Application) REF: pp. 394, 398

11. A Mexican immigrant mother tells the nurse she is concerned that people in the community will give her daughter mal de ojo. To which of the following folk illnesses is the nurse referring? A. Evil eye B. Fallen fontanel C. Soul loss D. Indigestion

ANS: A Evil eye A common folk illness that a nurse may encounter with the Mexican client is mal de ojo, or evil eye. DIF: Cognitive Level: Apply (Application) REF: p. 401

7. A nurse's Mantoux test is positive for exposure to tuberculosis. Which of the following conclusions should be drawn by the nurse? A. The nurse has been exposed to tuberculosis. B. The nurse has tuberculosis. C. The positive test result probably is due to a problem in the testing process. D. The test is inaccurate and needs to be repeated.

ANS: A The nurse has been exposed to tuberculosis. An individual who tests positive has been exposed and may be infected, but if that person shows no clinical signs, the person is not diseased. Infection refers to the entry, development, and multiplication of the infectious agent in the susceptible host. Disease is one of the possible outcomes of infection. People with latent TB have no symptoms, are not infectious, and can continue on with life. They may develop active TB. A nurse who interacts with clients may receive INH for a year as a precaution. DIF: Cognitive Level: Analyze (Analysis) REF: pp. 480-481

A nurse uses the Community Health Promotion Model to address the problem of increasing teenage pregnancy rates in the community. Which of the following steps would be used by the nurse? (Select all that apply.) a. Identifying interest in the topic through use of community forums b. Building a partnership with parents of teenagers to address this problem c. Counseling teenagers about effective birth control methods d. Contacting other cities with similar problems and issues e. Referring teenagers to Planned Parenthood

ANS: A, B The implementation of the Community Health Promotion Model includes identifying interest by community forums and building the partnership. Counseling and referring teenagers and contacting other cities may be used by the nurse but are not part of the steps identified by the Community Health Promotion Model. REF: pp. 450-451

A nurse would like to take action to reduce the risk of liability. Which of the following interventions should be used by the nurse? (Select all that apply.) a. Provide accurate documentation of client visits b. Inform clients of their rights of appeal c. Refer clients to the providers of their choice d. Utilize care planning whenever possible e. Promote strong community partnerships

ANS: A, B To reduce risk exposure, it is important to provide clear documentation of client encounters and inform clients of their rights of appeal. Client choice is important when choosing a provider; however, the nurse is responsible for using reasonable care when selecting referral choices for a client. Although care planning may be helpful for establishing a plan with the client, it is not necessary in reducing the risk of liability. REF: p. 497

17. A nurse's family has moved to a rural area to be near her spouse's aging parents. Which of the following would the nurse most likely notice about the family's new environment? (Select all that apply.) A. It is very difficult to obtain continuing education, except for online programs. B. Neighbors seem to expect the nurse to know everything about all health care and illness conditions. C. Neighbors ask questions about health concerns any time they see the nurse, regardless of where they are. D. People come to the health clinic before their situation becomes relatively serious.

ANS: A, B, C A. It is very difficult to obtain continuing education, except for online programs. B. Neighbors seem to expect the nurse to know everything about all health care and illness conditions. C. Neighbors ask questions about health concerns any time they see the nurse, regardless of where they are. Rural people often develop independent and creative ways to cope because of the distance, isolation, and sparse resources they encounter. The boundaries between home and work blur because clients are also neighbors and friends of immediate family members. Nurses are highly regarded by the community and viewed as experts on health and illness. Residents may ask health-related questions and request recommendations whenever they see the nurse. Nurses in rural areas may also be expected to, in general, know something about everything, and this can be a demanding expectation. Some of the challenges are professional isolation, limited opportunities for continuing education, lack of other health personnel or professionals with whom the nurse can interact, heavy workloads, the need to function well in several clinical areas, lack of anonymity, and for some, a restricted social life. DIF: Cognitive Level: Apply (Application) REF: p. 402

Which of the following adolescent(s) would receive care from the advanced practice nurse without parental consent? (Select all that apply.) a. A 16-year-old who is living on his own (not with his parents) b. A pregnant adolescent c. An adolescent in an emergency situation d. An adolescent whose diagnosis has a serious prognosis

ANS: A, B, C Most states have enacted laws allowing health care providers to treat adolescents in certain situations without parental consent. These situations include emergency care, substance abuse, pregnancy, and birth control. All 50 states recognize the "mature minors doctrine." This allows youths 15 years of age and older to give informed medical consent if it is apparent that they are capable of understanding

After seeing a public education program on the need for screening colonoscopy and the dangers of colorectal cancer, an older friend asks the nurse, "I'm really scared of getting cancer. What can I do to avoid that kind of cancer?" Which of the following recommendations should be made by the nurse? (Select all that apply.) a. Avoid smoking or much alcohol. b. Choose poultry or fish rather than red or processed meat. c. Eat lots of fruits, vegetables, and fiber each day. d. Try to get at least 8 hours of sleep a night.

ANS: A, B, C Obesity, physical inactivity, smoking, heavy alcohol consumption, a diet high in red or processed meats, and insufficient intake of fruits and vegetables are risk factors for colorectal cancer.

What principles were used in the creation of the Healthy Cities movement? (Select all that apply.) a. Primary care b. Equity in health promotion c. Community participation d. High technologic environments e. Relationship building

ANS: A, B, C The Healthy Cities movement was based upon the principles of primary care, equity in health promotion, and community participation. Relationship building and high technologic environments were not part of the creation of the Healthy Cities movement REF: p. 444

According to the Ottawa Charter for Health Promotion, which of the following are included in the areas for health promotion action? (Select all that apply.) a. Creating supportive environments b. Developing personal skills c. Building healthy public policy d. Reorienting health services

ANS: A, B, C, D Creating supportive environments, developing personal skills, building healthy public policy, and reorienting health services are all areas for health promotion action identified by the Ottawa Charter for Health Promotion in 1986. Developing community partnerships was not addressed by the Ottawa Charter for Health Promotion. REF: p. 442

What knowledge and skills are required in order to become a competent case manager? (Select all that apply.) a. Knowledge of community resources and financing mechanisms b. Written and oral communication skills c. Proficient negotiation and conflict-resolving practices d. Application of evidence-based practices and outcomes measurements e. Experience with ethical decision making

ANS: A, B, C, D The knowledge and skills required to achieve competency include knowledge of community resources and financing mechanisms, written and oral communication and documentation, proficient negotiating and conflict-resolving practices, and the ability to apply evidence-based practices and outcomes measures. Experience with ethical decision making is not part of becoming a competent case manager. REF: p. 481

A nurse requests to meet a newly referred family in their home. Which of the following best explains the rationale for this request? (Select all that apply.) a.The nurse can assess the family environment. b.The family will feel more comfortable. c.Families typically welcome others into their home. d.More family members can typically be involved

ANS: A, B, D Advantages to meeting in the family home include the fact that it allows the nurse to see the everyday family environment and observe typical family interactions. Also, more family members can be present, and families are often more comfortable in their own environment. However, a disadvantage to meeting in the family's home is that family members may view this as an intrusion into the only place they feel safe from outside observation; thus, the nurse must be highly skilled in guiding the interactions and setting limits.

Which of the following factor(s) may help determine how many home visits are made to a particular family? (Select all that apply.) a.Agency's policies regarding eligibility for services b.Family's feelings about the home visit and willingness to continue c.Nurse's perception of the amount of time needed to complete required tasks d.Reimbursement policies of third-party payers

ANS: A, B, D Although it is not unusual to have only one home visit with a family, often multiple visits are made. The frequency and intensity of home visits vary not only with the needs of the family but also with the eligibility of the family for services as defined by agency policies and priorities. Although the textbook does not directly discuss the issue, the family's willingness to work with the nurse is a factor. Also, the nurse cannot make visits unless the agency is being reimbursed for the nurse's time and expenses, so reimbursement policies of third-party payers are a major influence on the number of visits for which the family may be eligible. Thenurse's perception of the time needed to give quality care must unfortunately be secondary to other variables, which can control the time available

A family asks the nurse to please meet at their home rather than at the clinic. Which of the following best describes why the family prefers to meet in their home? (Select all that apply.) a.The family won't have to travel. b.It is cheaper for the family because of reimbursement requirements. c.Meeting at home is much more convenient for the family. d.The nurse won't be distracted by other clients or responsibilities. e.It would save money for the nurse and the clinic

ANS: A, C Advantages of a home visit include client convenience and client control, as well as the fact that it facilitates clients who are unable to travel, it allows more individualized services, and it provides a natural relaxed environment for discussion. However, home visits are expensive for the nurse and the nurse's employer because of travel costs and the amount of time spent with just one family. Unfortunately, nurses can be distracted by other tasks regardless of setting. Home visits are cheaper for insurance companies, not for the family

Which of the following interventions would the nurse most likely implement when addressing the problem of asthma among school-aged children? (Select all that apply.) a. Assess schools and day care centers for environmental "friendliness." b. Share nutritional information with all students in the school. c. Develop home and environmental assessment guides. d. Teach all school personnel how to use rescue inhalers.

ANS: A, C Population-focused strategies for asthma management include education programs for families of children and adolescents who have asthma, development of home and environmental assessment guides to identify triggers, education and outreach efforts in high-risk populations to aid in case finding (e.g., in areas with low income, high unemployment, and substandard housing, where there is exposure to secondhand smoke), development of community clean air policies (e.g., no burning of leaves, use of smoke-free zones), improved access to care for asthmatic patients (e.g., developing clinic services with consistent health care providers to decrease emergency department use), and assessment of schools and day-care centers for lack of asthma triggers.

A nurse's excessive attention to cost containment impairs the nurse's duty to provide measures to improve health. Which of the following ethical principles is being influenced? a. Autonomy b. Beneficence c. Veracity d. Nonmaleficence

ANS: B Beneficence is influenced when excessive attention to cost containment supersedes or impairs the nurse's duty to provide measures to improve health or relieve suffering. Autonomy is the individual's right to choose a provider. Veracity is truth telling which is important to building trusting relationships with clients. Nonmaleficence is doing no harm, which is addressed when incorporating outcomes measures, evidence-based practice, and monitoring processes in plans of care. REF: p. 497

16. A nurse tells her nursing supervisor that her family is moving from the urban area where they both live to a rural area to be near her spouse's parents, who are becoming less independent. Which of the following suggestions would the nursing supervisor provide to the nurse? (Select all that apply.) A. "Community members will probably hold you in higher regard and will look up to you." B. "Expect to have less autonomy in a small town than you have working as a nurse in a medical center." C. "You may feel like an isolated outsider, because the community may not immediately accept you." D. "You will have to be very sensitive about the differences in the rural lifestyle."

ANS: A, C, D A. "Community members will probably hold you in higher regard and will look up to you." C. "You may feel like an isolated outsider, because the community may not immediately accept you." D. "You will have to be very sensitive about the differences in the rural lifestyle." Nurses working in rural areas usually have a prestigious status in the community and are viewed as role models. They have greater (not less) independence and autonomy, fewer (not more) resources, and need to have more generalist (not specialized) knowledge and skills. DIF: Cognitive Level: Apply (Application) REF: p. 402

Which of the following must be firmly established before beginning a family assessment? (Select all that apply.) a.Why the data are needed b.How best to interview each individual in the family c.The most convenient time for you to visit the family d.The rationale or purpose of the visit

ANS: A, C, D Assessment of families requires an organized plan, including the purpose of seeing the family,which family members can be present, what you are assessing and why, and how will you obtain the necessary data. It can be assumed that the nurse would already know agency policies. It is more informative to interview the family as a whole so that you can observe family interaction (rather than focusing on interviewing each individual). Therefore, the preferred time to visit is when most family members will be available. Similarly, it is more informative to see the family in their home setting than to establish a different site for the visit.

14. Which of the following best describes why health professionals would be particularly concerned about the health needs of residents in rural areas? (Select all that apply.) A. About 25% of all U.S. residents live in rural settings. B. People in rural areas are especially susceptible to acute illnesses rather than chronic diseases. C. Diagnoses in rural areas are usually for physical injuries, not mental health concerns. D. A high prevalence of poverty exists among rural families.

ANS: A, D A. About 25% of all U.S. residents live in rural settings. D. A high prevalence of poverty exists among rural families. About 25% of all U.S. residents live in rural settings. Rural residents are less likely to engage in preventive behavior and they tend to be poorer than their urban counterparts (more than 25% of rural Americans live in or near poverty, and nearly 40% of all rural children are impoverished). Nearly one half of all rural adults suffer from at least one chronic condition. Stress, stress-related conditions, and mental illness are prevalent among populations that have economic difficulties, including rural populations. Often rural health professionals live and practice in a particular community for decades. Consequently, rural respondents tend to know their usual provider of health care. DIF: Cognitive Level: Remember (Knowledge) REF: pp. 391-392

A nurse calls a family to arrange for the first home visit. Which of the following information should the nurse share with the family? (Select all that apply.) a.The reason for the visit b.Everything the nurse knows about the family c.How many visits will be planned d.The cost of the visit and how this may be paid

ANS: A, D The nurse should include the reason for the visit, how or from whom the referral was obtained, and a brief summary of what is known about the family's situation. The nurse should negotiate a time for the visit, preferably when most family members are available. Clients should be told the fee and possible methods of payment before the nurse assesses the family's willingness for a home visit. If the family does not have a phone, mail can be used to share information.

A nurse is involved in implementing the Community Health Promotion Model. Which of the following best describes the action being taken by the nurse? a. Changing public policy b. Establishing a team to plan and coordinate the work c. Providing nursing diagnoses for the community d. Revising an objective for Healthy People 2020

ANS: B A structure in the community for health promotion should be developed through the establishment of a steering committee that will plan and coordinate the work. The nurse would not change public policy, but rather provide information based on data to policy makers. The nurse would not be revising objectives in Healthy People 2020 as that is not an action that individuals are able to take. Nursing diagnoses are not written as part of this model. REF: pp. 450-451

Which of the following best explains why some health clinics allow clients to be tested for HIV anonymously with no record of the client's name, address, or contact information? a.Client doesn't actually ever have to be told the results of the test. b.Client may be engaged in illegal activities (drug use). c.Client plans on not paying for the test and collection agencies will not be able to harass them. d.Client wants to be sure care providers don't share results with their family

ANS: B An advantage of anonymous testing may be that it increases the number of people who are willing to be tested, because many of those at risk are engaged in illegal activities. The anonymity eliminates their concern about the possibility of arrest or discrimination

A mother felt very guilty that her baby was born HIV positive. When the nurse suggested the usual DPT and MMR immunizations, the mother was extremely upset. "Don't you know HIV children are immunosuppressed?" she exclaimed. Which of the following would be the nurse's best response? a."All children have to have these immunizations before they can attend school." b."Being HIV positive, your child is more likely to catch an infection and be very ill if not immunized." c."I'm so sorry; I forgot for a moment your child was HIV positive." d."The American Pediatric Association requires all health care providers to offer these immunizations to all parents; it is your choice whether or not to accept them."

ANS: B Because of impaired immunity, children with HIV infection are more likely to get childhood diseases and suffer serious consequences of the diseases. Therefore, DPT, IPV, and MMR vaccines should be given at regularly scheduled times for children infected with HIV. Other immunizations may also be recommended after medical evaluation. Many states do have a "noshots, no school" law, but exceptions can be made. The APA does recommend immunizationsfor most children, but this is not the best answer.

An enduring process in which a manager establishes systems and monitors the health status, resources, and outcomes for a targeted aggregate of the population is called: a. case management. b. care management. c. disease management. d. demand management.

ANS: B Care management is an enduring process in which a manager establishes systems and monitors the health status, resources, and outcomes for a targeted aggregate of the population. Case management is defined as a collaborative process of assessment, planning, facilitation, care coordination, evaluation, and advocacy for options and services to facilitate an individual's and family's comprehensive health needs through communication and available resources to promote quality cost-effective outcomes. Disease management constitutes systematic activities to coordinate health care interventions and communications for populations with disease conditions in which client self-care efforts are significant. Demand management seeks to control use by providing clients with correct information and education strategies to make healthy choices, to use healthy and health-seeking behaviors to improve their health status, and to make fewer demands on the health care system. REF: p. 478

The nurse and client, a 20-year-old expectant mother, are discussing the advantages and disadvantages of breastfeeding. To enhance understanding of the implications of breastfeeding, the nurse says, "Tell me more about how you will work full time and breastfeed." Which of the following aspects of the advocacy process is being used? a. Verification b. Clarification c. Amplification d. Affirmation

ANS: B Clarification is a process in which the nurse and client strive to understand meanings in a common way. Verification is the process used by the nurse advocate to establish accuracy and reality in the informing process. Affirmation is based on an advocate's belief that a client's decision is consistent with the client's values and goals. Amplifying occurs between the nurse and the client to assess the needs and demands that will eventually frame the client's decision. REF: p. 491

A nurse utilizes the Centers for Disease Control and Prevention's (CDC's) Healthy Communities Program. Which of the following factors should the nurse recognize that will influence the continuance of this program? a. Time b. Community participation c. Federal involvement d. Technology

ANS: B Community participation can influence the continuance of the Healthy Communities and Cities movement. Time, federal involvement, and technology are not as major of factor to continuation as is participation by the community. REF: p. 449

A nurse is working as a case manager and is in the process of performing interdisciplinary, family and client conferences. Which of the following phases of the nursing process is being implemented? a. Assessment b. Diagnosis c. Planning for outcomes d. Implementation

ANS: B Diagnosis includes the identification of a problem/opportunity. Examples of activities used during the diagnosis phase include holding conferences, determining conclusions on the basis of assessment, and using interprofessional teams. Examples of assessment include developing networks with target populations and dissemination of written materials. Examples of planning for outcomes include validating and prioritizing problems and selecting evidence-based interventions. Examples of implementation include contacting providers and coordinating care activities. REF: p. 282 (Table 22-1)

Which of the following best describes where health care dollars in the United States be focused to improve breast cancer cure rates? a. Education for women about breast cancer b. Early detection programs with referral to ongoing access to a care provider c. Primary prevention programs d. Tertiary care through long-term follow-up

ANS: B Early detection can promote a cure, whereas late detection typically ensures a poor prognosis. The differences in the outcomes between women of color and white women point to issues associated with early detection, access to health care, and follow-up by a regular care provider.

The hospital-based nurse has worked with a client at some length regarding appropriate diet. Based on the family systems theory, which of the following will most likely occur when the client returns home? a.The family member who prepares food will probably suggest the newly discharged member eat the meals everyone in the family enjoys. b.The family member who prepares food will probably try to modify family meals without obvious change for the family as a whole. c.The family member who prepares food will probably prepare meals based on the diet plan for all the family. d.The family member who prepares food will probably prepare special meals for the newly discharged member

ANS: B Family systems typically maintain stable patterns, although families do change constantly in response to stresses. Change in one part of the family affects the total system. It is not realisticto expect the whole family to change eating patterns immediately based on the needs of one family member. However, if family members are supportive, they will want to try to help the ill member. Because of the rapid change and stress in American society, preparing different sets of meals is not very realistic. Therefore, the member who prepares the meals will probably compromise by trying to meet the ill member's needs without making drastic changes in the overall eating patterns of the family

Which of the following factors must be considered before deciding on an appropriate plan of action?a.Family agrees to the nurse's plan. b.Family is capable of the required actions. c.Family will learn better coping skills from the nurse's plan. d.Nurse has informed family how to complete the required actions

ANS: B Family theorists stress that any intervention plan must be developed in collaboration with the family, using and enhancing family strengths and increasing independence of family members. The plan cannot be the nurse's choice alone. Further, the plan must be within the information and skill level of the family, and the family must be committed to the plan and have adequate resources available to implement the plan

A case manager submits documentation that a nursing visit was completed at a client's home, but it was never performed. According to the general areas of legal risk, how would this action best be categorized? a. Liability for managing care b. Fraud/abuse c. Negligent referral d. Confidentiality/security

ANS: B Fraud/abuse occurs when false statements of claims are made and filed. Confidentiality/security occurs when sensitive information is not secured. Negligent referral occurs when referrals or treatments are inappropriate. Liability for managing care occurs when case management is not used appropriately, when there is harassment, or when care is inappropriately delegated. REF: pp. 496-497

Which of the following characteristics indicates a man is at a higher risk for developing prostate cancer? a. Being of Caucasian descent b. Has not had a PSA test c. Has a father or brother who has had prostate cancer d. Has benign prostatic hypertrophy (BPH)

ANS: C Having a father or brother who has had prostate cancer places a man at higher risk for developing prostate cancer.

Which of the following best describes the use of genomic health care? a.Assists with understanding family relationships b.Assists with determining familial health risks c.Useful in learning about environmental risk factors d.Useful in detecting risk for developing cancer

ANS: B Genomic health care can give health care providers the tools that they need to use a person's unique genomic information to design and prescribe the most effective treatment for each person and to help clients and families understand some of their health risks that are influenced by their genetic make-up. When nurses obtain a family history and learn about the illnesses and causes of death of biologically related family members, they can then learn aboutshared genes, environment and lifestyle behaviors that can increase a person's risks for the same diseases that other family members experienced. Genomic health care is broader than detecting risk for developing cancer

A nurse is examining the route of HIV transmission for a newly diagnosed HIV client. Which of the following would most likely be discovered by the nurse? a. Having contact with an HIV-positive individual who is coughing b. An infant receiving breast milk from an HIV-positive mother c. Receiving a mosquito bite while in Africa d. Being near an HIV-positive individual who is sneezing

ANS: B HIV can be transmitted through breast milk. HIV is not transmitted by coughing, sneezing, or mosquito bites.

In comparison with HIV infection in adults, HIV infection in infants and children: a. has the same signs and symptoms. b. has a shorter incubation period. c. has a longer survival period. d. is detected by using the same tests.

ANS: B HIV infection in infants and children has a shorter incubation period. The physical signs and symptoms in children are different and include failure to thrive, unexplained persistent diarrhea, developmental delays, and bacterial infections such as tuberculosis (TB) and severe pneumonia. Detection is made through different tests in infants of infected mothers than from those who are over 18 months. The effectiveness of antiretroviral therapy in pregnant women and newborns in preventing transmission from mother to fetus or infant has made pediatric HIV rates decline sharply; thus, these children are less likely to contract the virus.

Which of the following best describes the characteristic appearance of lesions of human papillomavirus (HPV)? a.Solitary growth with elevated borders and a central depression b.Elevated growths with a "cauliflower" appearance c.Thin-walled pustules that rupture to form honey-colored crusts d.Vesicles that ulcerate and crust within 1 to 4 days

ANS: B HPV causes genital warts that appear as textured surface lesions, with what is sometimes described as a cauliflower appearance. The warts are usually multiple and vary between 1 and 5 mm in diameter

The nurse is told that a healthy, functional family consisting of a 25-year-old man and a 24-year-old woman, who are expecting their first child, would appreciate a nurse coming to their apartment for anticipatory guidance in preparing themselves and their apartment for the baby. Based on that statement, which of the following assumptions can the nurse safely make about the family? a.The family is lacking a strong support system. b.The family's basic needs are being met. c.The couple's in-laws are unavailable to share their expertise about child care. d.The married couple is excited about their first baby

ANS: B In functional, healthy, or resilient families, the basic survival needs are met. Healthy families exist based on attachment and affection. There is nothing in the example to suggest that they are married, that their income is low, or that they lack other resources or support systems

A client with tuberculosis (TB) asks why the nurse is required to watch the client swallow the medication each day. Which of the following statements is the best response by the nurse? a."Clients with TB are often noncompliant, so if I directly observe, you will be sure to take the drugs that have been ordered." b."This therapy is recommended to make sure that you receive the treatment you need and the infection doesn't become resistant to the drugs." c."This is to make sure you take your medication if your condition becomes so advanced that you do not have enough cerebral oxygenation to remember." d."Tuberculosis medications are very expensive so this method ensures that government money doesn't get wasted on those who will not take the drugs."

ANS: B It is important to be respectful to clients and to consider their perspective and psychological health while also responding truthfully. Directly observed therapy (DOT) programs for TB medication involve the nurse observing and documenting individual clients taking their TB drugs. When clients prematurely stop taking TB medications, there is a risk that the TB will become resistant to the medications. This can affect an entire community of people who are susceptible to this airborne disease. DOT ensures that TB-infected clients receive adequate medication. Thus, DOT programs are aimed at the population level to prevent antibiotic resistance in the community and to ensure effective treatment at the individual level. Many health departments have DOT home health programs to ensure adequate treatment

Which of the following best explains why chlamydia is a major focus of public health efforts? a.It has more serious long-term outcomes than other STDs b.It can cause problems in infants born to infected mothers c.It is not frequently seen in the United States d.It is so difficult and expensive to treat

ANS: B Like gonorrhea and other STDs, chlamydia can cause neonatal complications in infants born to infected mothers, although it is rather easily treated with antibiotics. Chlamydia does have serious long-term outcomes for the client, but so do syphilis and other STDs. However, unlikesyphilis, which in its later stages is rare in the United States, chlamydia is the most common reportable infectious disease in the United States and hence is a major focus of public health

An occupational health nurse is developing an educational program to address the importance of healthy personal health habits. Which of the following topics would be most important for the nurse to address? a.Avoidance of alcohol b.Regular physical exercise c.Daily consumption of calcium-rich foods d.Monthly self-breast and testicular exams

ANS: B Many family health risks can be reduced by careful attention to diet, exercise, and stress management. Regular physical exercise is effective in promoting and maintaining health and in preventing disease. Physical activity can help to prevent obesity, diabetes, heart disease, cancer, osteoporosis, and depression. Avoidance of alcohol and daily consumption of calcium-rich foods are not recommendations for improving personal health habits

Which of the following is of the greatest concern for the nurse who works with high school students? a. Alcohol use b. Motor vehicle accidents c. Sports-related injuries d. Unprotected sex

ANS: B Motor vehicle-related injuries and violence are the leading causes of morbidity and mortality for adolescents. Thus, this would be the greatest concern for the nurse working with high school students.

Which of the following should be the initial consideration made by a nurse who is working with lesbian, gay, bisexual, and transgendered (LGBT) families? a.Understanding of same sex marriage laws within the state b.Understanding of personal feelings of working with members of this community c.Assessment of the family structure within the LGBT family d.Assessment of sexual orientation in a safe environment

ANS: B Nurses have an ethical obligation to provide culturally competent care to LGBT families. Some nurses may feel a degree of discomfort discussing sexual orientation with their patients. However, it is important to overcome this barrier to care for LGBT families. Thus, nurses should provide a safe environment for patients to discuss their sexual orientation

A nurse is using population management as part of the nursing care that is provided. Which of the following activities is the nurse most likely completing? a. Census taking to determine the total number of people in the population b. Assessing the needs of the client population through compilation of health histories c. Providing case management services for every citizen in the community d. Selecting programs for wellness that are repeated annually

ANS: B Population management involves assessing the needs of the client population through health histories, claims, use-of-service patterns, and risk factors. Population management does not include census taking, providing case management services for all, or repeating wellness programs on an annual basis. REF: p. 477

A nurse was preparing for a home visit to a family where the mother had just been discharged from trauma care after being hit by a drunk driver. The nurse hoped the family was able to care for her. Which of the following comments from the husband would suggest an energized family? a."I make most of the decisions so the kids don't notice much difference." b."My daughter is pretty independent; she's active in both sports and theater." c."My son is old enough to get a job and help pay all these medical bills." d."My wife taught our daughter how to cook simple meals."

ANS: B Pratt proposed the energized family as being an ideal family type that was most effective in meeting health needs. The energized family is characterized by active contact with a variety ofgroups and organizations (Boy Scouts, church, sports, theater), flexible role relationships (not if only the daughter is taught how to cook and only the son is expected to get a job), equal power structure (not if the husband makes all the decisions), and a high degree of autonomy by each member (kids know how to be flexible, because parents travel for business frequently).

A nurse is teaching a client diagnosed with gonorrhea how to prevent reinfection and further spread. Which of the following describes the action taken by the nurse? a. Primary prevention b. Secondary prevention c. Tertiary prevention d. Primary health care

ANS: B Secondary prevention focuses on early detection and prompt treatment of disease, injury, or disability. Primary prevention refers to those interventions aimed at preventing the occurrence of disease, injury, or disability. Tertiary prevention focuses on chronic care and rehabilitation. Primary health care refers to the first line of care provided to patients typically by a physician or other health care provider.

A nurse upholds a client's right to make a choice and to act on the choice. Which of the following best describes the action being taken by the nurse? a. Affirming b. Supporting c. Informing d. Amplifying

ANS: B Supporters approve and support clients' actions. Affirming is based on an advocate's belief that a client's decision is consistent with the client's values and goals. Informing includes educating clients about the nature of their choices, the content of those choices, and the consequences to the client and is not a one-way activity. Amplifying occurs between the nurse and the client to assess the needs and demands that will eventually frame the client's decision. REF: p. 491

Which are examples of knowledge domains used in case management? a. Legal issues, malpractice recognition, and community involvement b. Teaching, counseling, and education skills c. Advocacy, political campaigning and legislative change d. Grant application, bargaining contracts, and securing funding

ANS: B Teaching, counseling, and education skills are an important part of case management in addition to program evaluation and research. The other domains listed are not used in case management REF: p. 484 (Box 22-2)

A nurse is concerned about the prevalence of tuberculosis among migrant farmworkers. Which of the following activities would be best to use when implementing tertiary prevention? a.Administer purified protein derivative (PPD) to contacts of those with tuberculosis. b.Initiate directly observed therapy (DOT) for tuberculosis treatment. c.Provide education about the prevention of tuberculosis to members of the migrant community. d.Use skin tests to screen migrant health workers for tuberculosis infection

ANS: B Tertiary prevention is carried out among persons already infected with the disease. In this instance DOT ensures compliance with treatment to cure the disease and to prevent worseningor the development of secondary problems

A case manager provides a formal communication link among all parties concerning the plan of care management. Which of the following roles of the nurse is being implemented? a. Facilitator b. Liaison c. Coordinator d. Negotiator

ANS: B The case manager providing a formal communication link among all parties concerning the plan of care management is performing the role of liaison. As a facilitator, the nurse supports all parties in work toward mutual goals. When working as a coordinator, the case manager arranges, regulates, and coordinates needed health care services for clients at all necessary points of services. The case manager who uses effective collaboration and team strategies to make arrangements for services is performing the role of negotiator. REF: p. 484 (Box 22-1)

A case manager uses effective collaboration and team strategies to make arrangements for services. Which of the following roles is being applied? a. Broker b. Negotiator c. Liaison d. Facilitator

ANS: B The case manager who uses effective collaboration and team strategies to make arrangements for services is performing the role of negotiator. The role of broker is used when acting as an agent for provider services that are needed by clients to stay within coverage according to budget and cost limits of the health care plan. When acting as a liaison, the nurse provides a formal communication link among all parties concerning the plan of care management. As a facilitator, the nurse supports all parties in work toward mutual goals. REF: p. 484 (Box 22-1)

A nurse is providing contraceptive counseling to a female client. Which of the following is the most appropriate outcome of this counseling? a. Encourage the individual to choose abstinence. b. Ensure the individual is educated to make an informed choice about reproduction. c. Advocate for increased funding for reproductive services. d. Reduce the health risks of the individual.

ANS: B The goal of contraceptive counseling is to ensure that women have appropriate instruction to make informed choices about reproduction. The nurse should provide a nonjudgmental approach during counseling and allow the woman to choose the appropriate contraceptive method. Nurses do advocate for reproductive services for women, but that is not a goal of contraceptive counseling. Reduction in the health risks of the individual is a goal of pre-conceptual counseling.

A nurse is working with a family who is confronting major challenges to their health. Which of the following approaches would be most helpful for the nurse to use? a.Allowing the family to be noncompliant b.Building on the family's strengths and resilience c.Labeling the family as resistant d.Recognizing that the family is dysfunctional

ANS: B The labels of dysfunctional, noncompliant, resistant, or unmotivated all denote families who are not functioning well; however, such labels do not create an environment conducive for positive family change and intervention and should not be used. Families are neither all good nor all bad; families have both strengths and difficulties and have seeds of resilience. Recognizing the family's strengths gives the nurse assets on which to draw in planning care

When a nurse discovers that a woman has been treated for cervical cancer, the nurse asks the woman whether she has ever been tested for HIV or other STDs. The woman is offended and asks why the nurse would ask her such a thing. Which of the following statements would be the best response from the nurse? a."Cervical cancer treatments may decrease immunity, so that it is easier to acquire STDs." b."Cervical cancer usually is caused by HPV, and often the presence of one STD is accompanied by other STDs." c."The presence of an STD in women with cervical cancer may lead to congenital defects in offspring." d."The presence of an STD in a woman with a history of cervical cancer has been associated with a relapse of the cancer after treatment."

ANS: B The link between HPV infection and cervical cancer has been established and is associated with specific types of the virus. In 80% to 90% of cases of cervical cancer, evidence of HPV has been found in the tumor. Additionally, HSV-2 infection is linked with the development of cervical cancer. Because the presence of an STD increases the risk for the presence of other STDs, it is essential to screen for this information

The nurse is involved in a conflict resolution situation with the parents of a 2-year-old boy. The parents are deciding if "spanking" the child is a disciplinary method that they will employ. The mother says, "I do not believe in spanking. I see it as abusive and demeaning." Which of the following best describes the statement by the mother? a. Cooperation b. Assertiveness c. Bargaining d. Collaboration

ANS: B The mother's statement is an example of assertiveness, the ability to present one's own needs. Cooperation is the ability to understand and meet the needs of others. Bargaining is part of the negotiation process when debates include gathering facts based on reasoning that will generate understanding and promote relearning. Collaboration is a dynamic highly interactive and interdependent process in which people work together to share resources and a vision for a goal. REF: p. 494

Which model addresses the structure and processes of using the population-based tools of disease management and critical pathways to offer care for client populations? a. Client-focused b. System-focused c. Social service d. Long-term care

ANS: B The system-focused model addresses the structure and processes of using the population-based tools of disease management and critical pathways to offer care for client populations. The concern in client-focused models is with the relationship between case manager and client to support continuity of care and to access providers of care. The social service models provide services to clients to assist them in living independently in the community and in maintaining their health by eliminating or reducing the need for hospital admissions or long-term care. Long-term care is not a model of case management. REF: p. 488

A nurse is utilizing the provisions created by the Ryan White HIV/AIDS Treatment Extension Act. Which of the following interventions is the nurse most likely performing? a. Increasing AIDS awareness in the community b. Determining available health care services for HIV-infected individuals c. Preventing the transmission of AIDS to children from their mothers d. Allowing persons in the final stages of HIV to die with dignity

ANS: B This act provides emergency services, services for early intervention and care, and drug reimbursement programs for HIV-infected individuals.

A nurse is educating intravenous (IV) drug users about sharing equipment. Which of the following information would be appropriate for the nurse to include in this education? a. Tell the clients to throw away their equipment after one use b. Educate the clients on using full-strength bleach on their drug paraphernalia for 30 seconds c. Suggest limiting the number of people who share the equipment d. Provide clean needles and syringes to whoever wants them

ANS: B Using bleach on the needles and syringes is a way to decrease cross-contamination. This is the last-resort option. People who inject drugs are difficult to reach for health care services, so providing them education is important so that they can protect themselves and others as they most likely will not throw away their equipment, or be selective about whom they share their equipment with. Providing needles and syringes does not provide the appropriate education to prevent the spread of disease.

Which part of the immunological system suffers the greatest damage as a result of HIV infection? a. Dendrite cells b. CD4+ T-lymphocytes c. Macrophages d. Monocytes

ANS: B When HIV enters the body, a person may experience a mononucleosis-like syndrome, referred to as a primary infection, which lasts for few weeks. This may go unrecognized. The body's CD4 white blood cell count drops for a brief time when the virus is most plentiful in the body. The dendrite cells, macrophages, and monocytes are not as severely damaged as the CD4 cells.

19. A student complains to the college health nurse that her academic work has been going downhill because of lack of sleep. "My 3-year-old probably misses her babysitter since she has started going to the big daycare center. She hasn't been sleeping well and keeps scratching her bottom. Hopefully, she'll adapt to daycare soon." Which of the following information should the nurse provide to the student? A. "Dry skin in winter weather can cause itchiness; try to put on lotion before bedtime." B. "Your daughter may have pinworms; let me teach you how to check for this." C. "Perhaps your child is not developmentally ready for group play." D. "Try to arrange more one-on-one time with your 3-year-old."

ANS: B "Your daughter may have pinworms; let me teach you how to check for this." Enterobiasis (pinworm infection) is the most common helminthic infection in the United States with about 42 million cases a year. This infection is seen most often among children in institutional settings. Pinworms cause itching, especially around the anus, which can result in a lack of sleep for both child and caregiver. DIF: Cognitive Level: Apply (Application) REF: p. 496

10. A nurse is working with Mexican immigrants. Which of the following behaviors would most likely lead to a positive interaction for the nurse? A. Avoiding touching the client except when necessary as part of the physical examination B. Calling the client by name, socializing before addressing the problem, and being very respectful C. Keeping all interactions direct, to the point, and targeted on the reason for presentation D. Maintaining a non-confrontational relationship by avoiding any disagreement even if the nurse does disagree with what the client is saying

ANS: B Calling the client by name, socializing before addressing the problem, and being very respectful The nurse is considered an authority figure who should respect (respeto) the individual, be able to relate to the individual (personalismo), and maintain the individual's dignity (dignidad). Such an approach would be expected for any client, not just with immigrants. Mexican individuals, like those of many cultures, expect to establish some rapport through talking about personal matters (chit-chat) for the first few minutes of an encounter before addressing any problems. Touching as a caring gesture is usually seen as a positive behavior. DIF: Cognitive Level: Apply (Application) REF: pp. 400-401

5. Which of the following components of the epidemiologic triangle contributes most to a female client developing a vaginal infection caused by fungi after successful treatment of her strep throat with antibiotics? A. Agent B. Environment C. Host D. Agent and host

ANS: B Environment The antibiotic therapy eliminates a specific pathological agent, but it also may alter the balance of normally occurring organisms in the woman's body, which causes a change in the vaginal environment and allows normally present fungi to proliferate, resulting in a yeast infection. DIF: Cognitive Level: Analyze (Analysis) REF: p. 480

1. Which of the following is the number one cause of death worldwide? A. Chronic diseases (heart disease, cancer, stroke) B. Infectious diseases C. Injuries (accidental or purposeful) D. Terrorism

ANS: B Infectious diseases In countries with higher standards of living, where people live longer, chronic diseases—heart disease, cancer, and stroke—are the leading causes of death. Infectious diseases, however, are still the number-one cause of death worldwide. DIF: Cognitive Level: Remember (Knowledge) REF: p. 477

16. A student comes to the college health clinic with typical cold symptoms of fever, sneezing, and coughing, but the nurse also notes small white spots on the inside of the student's cheeks. Which of the following actions should be taken by the college health nurse? A. Inform all students, staff, and faculty of a possible rubella epidemic B. Inform all students, staff, and faculty of a possible measles epidemic C. Reassure the student that it is just a bad cold and will soon pass D. Tell the student to take two acetaminophen and drink lots of fluids

ANS: B Inform all students, staff, and faculty of a possible measles epidemic Measles is an acute, highly contagious disease that, although considered a childhood illness, is often seen in the United States in adolescents and young adults. Symptoms include fever, sneezing, coughing, conjunctivitis, small white spots on the inside of the cheek (Koplik spots), and a red, blotchy rash beginning several days after the respiratory signs. Measles is serious. Around 10% of measles cases require hospital admission. It can lead to pneumonia and encephalitis, and it can kill. Persons who may have been exposed should be informed that anyone under 18 who has not received both immunization doses should receive measles vaccine. DIF: Cognitive Level: Apply (Application) REF: p. 489

12. Which of the following conditions of the rural environment provides increased opportunities for teaching? A. Increased interaction among residents due to neighbors visiting neighbors on the family farms B. Involvement in rural community activities provides more contact with community residents than in urban areas C. Nursing responsibilities in these areas stress the importance of primary, secondary, and tertiary prevention D. Increased illnesses and injuries of rural residents require that they see nurses more often

ANS: B Involvement in rural community activities provides more contact with community residents than in urban areas Rural areas have fewer churches, stores, and schools. Thus, nurses may go to the same church, shop at the same stores, and/or have children in the same schools as their clients. There are also fewer organizations, so it is easier to become involved and meet everyone while participating. Visiting neighbors does not increase opportunities for teaching. The fact that nursing responsibilities include all levels of prevention is true of both urban and rural areas. The statement about increased illness and injuries in rural residents is wrong, because although illnesses and injuries are increased, rural residents often do not seek care for these conditions. DIF: Cognitive Level: Understand (Comprehension) REF: p. 402

20. Which of the following is the most common vector-borne disease worldwide? A. Dengue B. Malaria C. Onchocerciasis (river blindness) D. Yellow fever

ANS: B Malaria Globally, malaria is the most prevalent vector-borne disease, with over 2.4 billion people at risk and more than 275 million cases reported each year. More than 1 million children die of malaria each year. Dengue is the second most common vector-borne disease. DIF: Cognitive Level: Understand (Comprehension) REF: p. 497

13. Which of the following best describes the current goal in relation to communicable diseases? A. To control political borders so diseases cannot spread further B. To exterminate specific infectious agents one by one C. To expand health care facilities to improve infectious disease treatment D. To achieve worldwide immunization to control new cases

ANS: B To exterminate specific infectious agents one by one The goal of prevention and control programs is to reduce the prevalence of a disease to a level at which it no longer poses a major public health problem. In some cases, diseases may even be eliminated or eradicated. The goal of elimination is to remove a disease from a large geographical area (e.g., a country or region of the world), such as has been done with polio in the Americas. Eradication is the irreversible termination of all transmission of infection by extermination of the infectious agents worldwide, as has been done with smallpox. DIF: Cognitive Level: Understand (Comprehension) REF: p. 486

15. A nurse is caring for Mexican migrant farmworkers. Which of the following conditions are of greatest importance for the nurse to assess? (Select all that apply.) A. Cholera B. Hepatitis C. High blood level of lead D. Malaria

ANS: B, C B. Hepatitis C. High blood level of lead Poor quality and crowded housing can contribute to such health problems as tuberculosis (TB), gastroenteritis, and hepatitis, as well as exposure to high levels of lead. Tuberculosis (TB) is more prevalent in Mexico than in the United States, and some infected immigrants work as migrant farmworkers. Additionally, TB tends to spread in regions where housing is crowded. Migrant farmworkers have higher rates of TB than most population groups. DIF: Cognitive Level: Apply (Application) REF: p. 398

A nurse's mother leans forward and says, "My best friend fell the other day and now she's in the hospital. I'm really worried about getting osteoporosis. What do you think I should do?" What should the nurse recommend? (Select all that apply.) a. Ask your doctor for hormone replacement therapy. b. Continue to abstain from alcohol and not use tobacco. c. Eat foods high in calcium such as fortified skim milk. d. Eat lots of green leafy vegetables.

ANS: B, C It is estimated that one of every two American women older than 50 years will experience an osteoporosis-related fracture in her lifetime. Primary prevention activities include a diet rich in calcium and vitamin D; exposure to sunlight for 20 minutes a day; exercise, especially weight-bearing activities such as walking, running, stair climbing, and weight lifting, to improve bone density; limiting alcohol consumption; and avoiding smoking.

24. Six students order meals at a local restaurant. Which of the following students are at highest risk for illness? (Select all that apply.) A. The first student asks for a salad with chicken strips and dressing on the side. B. The second student asks for a hamburger, very rare. C. The third student orders a tuna salad sandwich with extra mayonnaise. D. The fourth student orders a breakfast meal with two very soft-poached eggs and toast.

ANS: B, D B. The second student asks for a hamburger, very rare. D. The fourth student orders a breakfast meal with two very soft-poached eggs and toast. Escherichia coli O157:H7 can produce a strong cytotoxin that can cause a potentially fatal hemorrhagic colitis. Hamburger is often involved in outbreaks, since the grinding process exposes pathogens on the surface of the whole meat to the interior of the ground meat, effectively mixing the once-exterior bacteria thoroughly throughout the hamburger so that searing the surface no longer suffices to kill all bacteria. Also, hamburger is often made of meat ground from several sources. The best protection against foodborne pathogens is to thoroughly cook food before eating it. Salmonella is also transmitted by undercooked foods such as eggs, poultry, dairy products, and seafood. Consequently, students eating very rare hamburger and undercooked eggs are at high risk. DIF: Cognitive Level: Apply (Application) REF: pp. 493-494

26. The advanced practice nurse explains that the client has an upper respiratory infection (URI) and suggests several measures that might make the client more comfortable. Which of the following best describes why the nurse doesn't just prescribe antibiotics as the client repeatedly requests? (Select all that apply.) A. Antibiotics are expensive, whereas the support measures would be almost free of cost. B. Viral diseases are not affected by antibiotics. C. Clinics cannot afford to continually give antibiotics to anyone who asks for them. D. The more antibiotics are prescribed, the more infectious agents develop resistance to such drugs.

ANS: B, D B. Viral diseases are not affected by antibiotics. D. The more antibiotics are prescribed, the more infectious agents develop resistance to such drugs. Antibiotics are not effective against viral diseases, a fact found unacceptable to many clients looking for relief from the misery of a cold or flu. The inappropriate prescribing of antibiotics contributes to the growing problem of infectious agents that have developed resistance to once-powerful antibiotics. DIF: Cognitive Level: Apply (Application) REF: p. 477

A nurse enters a family's home for the first time. Which of the following goals should the nurse have? (Select all that apply.) a.Assessing each family member in detail both physically and psychologically b.Collaborating with the family to establish goals and a plan for meeting them c.Determining the exact relationship between each member of the family d.Exploring the family's perception of their problems and needs

ANS: B, D During the beginning phase of the nurse-family interaction, three activities occur—mutual data collection and exploration of needs and problems; mutual establishment of goals; and mutual development of a plan

A nurse is using a critical path when providing care for a client. Which of the following most likely describes the situation that the nurse is in? a. Implementing the six "rights" of case management b. Demonstrating competencies required for practicing case management c. Attempting to achieve a measurable outcome for a specific client d. Effectively managing conflict resolution

ANS: C A critical path is a case management tool used to achieve a measurable outcome for a specific client case. The critical path details the essential and sequential activities in care, so that the expected progress of the client was known at a point in time. A critical path does not address the six "rights" of case management or demonstrate the competencies required for practicing case management. A critical path is not used for conflict resolution. REF: p. 485

A nurse is completing a health risk appraisal with a client. Which of the following comments would cause the nurse to probe further to determine if the family is in crisis? a."I can't visit my husband in the hospital when I'm at work all day. How can I be sure he's all right?" b."My husband always handled our finances. Now that he's gone, I'll have to learn how to do this." c."I don't know what to do now that my husband is dead. There is no way I can go back to work and also take care of our three children." d."What am I supposed to do now that everything we own is gone? Are there any agencies that can help me?"

ANS: C A family crisis occurs when the family is not able to cope with an event and becomes disorganized or dysfunctional. When the demands of the situation exceed the resources of the family, a family crisis exists. In three of the options, the survivor is considering the problem and trying to learn how to cope or seek resources to cope. Only in this option is the person overwhelmed and unable to conceive of how to cope

Which of the following families is at high risk for health problems? a.A man agrees that he needs to eat better and exercise more but also expresses how busy he is at his job. b.A man knows that his grandfather, father, and older brother all died of cardiac disease. c.A man is currently unemployed and despairs about finding a position. d.A man expresses disappointment that, having been laid off as an executive, his new position pays only about two thirds of his original salary

ANS: C A person who is unemployed and despairing of finding employment is at serious economic risk, which is one of the foremost predictors of health problems. Economic risk is determined by the relationship between family financial resources and the demands on those resources. Having adequate financial resources means that a family is able to purchase the necessary services and goods related to health, such as adequate housing, clothing, food, education, and health or illness care.

A person diagnosed with syphilis presents with signs and symptoms of rash, sore throat, and muscle and joint pain. Which of the following stages of syphilis is the client most likely experiencing? a. Congenital b. Primary c. Secondary d. Tertiary

ANS: C A person with signs and symptoms of a rash, sore throat, and muscle/joint pain is experiencing the secondary stage of syphilis. Primary syphilis occurs when the bacteria produce infection in the form of a chancre at the site of entry. Tertiary syphilis usually occurs several years after initial infection and is rare in the United States because the disease is usually cured in its early stages with antibiotics. Congenital syphilis is transmitted transplacentally.

A nurse is leading a team responsible for evaluating the health of a community using the Community Health Promotion Model. Based on this model, which of the following steps would you expect the nurse to complete? a. Ensuring high technological access before the plan can be implemented b. Soliciting the consent of every member of the community c. Assessing the community d. Restructuring the public health policy in the community

ANS: C Assessing the community is one of the nine steps of the Community Health Promotion Model. Restructuring public health policy, soliciting consent of all community members, and ensuring high technological access before implementing the plan are not part of this model. REF: p. 451

A nurse has just met a family and is being doing their family assessment. Which of the following actions should the nurse take before engaging in self-disclosure? a.Confirm the reason for the assessment. b.Demonstrate culture awareness. c.Take time to build trust. d.Understand the family dynamics

ANS: C Assessment is interactive. As you are evaluating families, they are evaluating you. Too much disclosure during the early contacts between the family and nurse may scare the family away. Slow the process down, and take time to build trust

Which of the following sexually transmitted diseases can be prevented through immunization? a.Chlamydia b.Gonorrhea c.Hepatitis B d.Herpes

ANS: C At the time of this writing, HBV infection can be prevented by immunization; vaccines for the remaining options are not yet available

A client has designated someone else to make health care decisions when he or she is unable to do so. Which of the following methods is the client using to make health care decisions? a. An advance directive b. A living will c. A durable medical power of attorney d. The Patient Self-Determination Act

ANS: C Durable medical power of attorney is the legal way for a client to designate someone else to make health care decisions when he or she is unable to do so.

A home health nurse is preparing to terminate the first home visit with teenage parents and their new baby. Which of the following actions will the nurse take before leaving? a.Determine the family's willingness for another home visit b.Establish the purpose of the visit c.Review the family's learning and other accomplishments of the visit d.Review the family record and reason for referral

ANS: C During the termination phase, the nurse reviews the visit with the family, summarizes what has occurred and what has been accomplished, and may make plans for future visits. The other options listed occur during the pre-visit phase

The wearing of seat belts is required in many communities. Children of a certain age must be restrained in a car seat or seat belt when riding in a car. Which of the following best describes what has happened in this community? a. Unilateral action of lawmakers b. Participation in health fairs c. Healthy public policy d. Media interest in protecting the public

ANS: C Examples of healthy public policy are seat belt legislation, nonsmoking policies, and immunization policies for school-age children. Participation in health fairs involves providing health education and resources out in the community. Unilateral action of lawmakers does not demonstrate community involvement. Media interest does not create the legislation. REF: p. 451

A nurse focuses on the care of the individual while viewing the client's family as a background resource or possible stressor. Which of the following conceptualizations of family does this nurse's view represent? a.Client b.Component of society c.Context d.System

ANS: C Family as the context, or structure, has a traditional focus that places the individual first and the family second. In the "family as context" concept, the family serves as either a resource ora stressor to individual health and illness

In taking a family history, the nurse in community health finds that this is the second marriage for the previously divorced parents and that the male partner is the stepparent to the oldest child. For which of the following aspects of the family assessment is data being gathered? a.Dynamics b.Function c.Structure d.System

ANS: C Family structure refers to the characteristics and demographics (gender, age, number) of individual members who make up the family. Structure defines the roles and the positions of family members

A nurse is new to the community but wants to begin planning immediately for health promotion programs. Even though the nurse does not yet know the community, which of the following programs would be good to plan first? a. Alcohol and tobacco cessation programs b. Cancer screening programs c. Cardiac health education programs d. Exercise for life programs

ANS: C Heart disease is one of the most significant public health problems in the United States, responsible for premature mortality and disability. Cardiovascular disease is the leading cause of death in the United States. Good nutrition and exercise programs, while both good health promotion programs, are too narrow in focus to represent the greatest need in the community. Cancer is the second leading cause of death so may be the nurse's second focus.

A school nurse wants to decrease the rate of obesity among children. Which of the following actions in the community would be most effective? a. Lobby legislators to enact stronger legislation regarding school lunches and snack machines in schools. b. Increase nutrition programs in schools that teach children to make healthy food choices. c. Involve the entire family in the planning and managing of nutrition, especially when a child in the family is obese. d. Provide after-school and summer camps that focus on diet and exercise.

ANS: C Interventions need to be based on goals of lifestyle changes for the entire family. The goal is to modify the way the family eats, exercises, and plans daily activities. Although it is important to teach nutrition, exercise, and proper food choice, if the family does not, for example, prepare the proper foods from which they can choose, the knowledge of the child is insignificant.

A nurse is using life care planning when working with a client. Which of the following would be the most appropriate time for this to be used? a. When organizing a timeline of life events b. When documenting client information and requests c. When assessing present and future client needs d. When estimating future costs for medical care

ANS: C Life care planning is a customized, medically based document that provides assessment of present and future needs. Typically, a life care plan incorporates medical, financial, psychological, vocational, built environment, and social costs during the remaining life of the client. Life care planning is a tool used in case management. A life care plan assesses the current and future needs of a client for catastrophic or chronic disease over a life span. REF: p. 486

Which of the following best describes a normative life event that can increase the risk for illness? a.A family is involved in a motor vehicle crash. b.A group of teens experiment with recreational drugs. c.A woman is pregnant with her first child. d.The family wage earner is laid off from his job

ANS: C Life events can increase the risk for illness and disability. Normative events are those that are generally expected to occur at a particular stage of development or of the life span. Although pregnancy is a normal condition, it carries risks such as the development of eclampsia or more minor health problems such as constipation and hemorrhoids. Additionally,pregnancy (and the birth that follows) will alter family dynamics and may increase risk for psychological stressors. The events listed in the other options are not normative life events

A nurse is drawing a genogram. Which of the following would the nurse use to demonstrate a marriage relationship between two individuals? a.A broken line b.A dashed line c.A solid line d.Two parallel lines

ANS: C Marriage is indicated by a solid line on a genogram

A mother says, "My son wants me to let him buy a car. I must admit, I certainly get tired of driving him around to all his sports and other activities. Do you think I should let him have a car?" Which of the following statements would be the best response by the nurse? a. "Absolutely, it will help him recognize the cost of gas and maintenance." b. "Certainly. Most young men want their own car, and peer pressure can be painful." c. "It depends on whether you trust your son to drive safely." d. "No, adolescent males cannot be trusted with an automobile."

ANS: C Motor vehicle-related injuries and violence are the leading causes of morbidity and mortality for adolescents. Males are more likely to take risks, and injury death rates for boys are twice as high as those for girls. Thus, the response by the nurse should take these factors into consideration in response and encourage the mother to individualize her decision for her son.

Mutual benefit with limited loss for everyone is a goal of: a. negotiating. b. assertiveness. c. conflict management. d. cooperation.

ANS: C Mutual benefit with limited loss for everyone is a goal of conflict management. This involves using skills directed toward learning all parties' needs and desires, detecting their areas of agreement and disagreement, determining their abilities to collaborate, and assisting in discovering alternatives and activities for reaching a goal. The process of moving conflicting parties toward an outcome is called negotiation. Assertiveness is the ability to present one's own needs. Cooperation is the ability to understand and meet the needs of others. REF: p. 493

Which of the following clients would cause the nurse the most concern? a.The client who is currently unemployed but actively seeking a position and frequently walking from one interview to another b.The client who is not employed but spending time at the gym keeping fit and studying the benefits of organic natural uncooked foods c.The client who is employed and often works 12 hours a day without moving from the computer desk d.The client who is employed but always leaves promptly at 5:00 to pick up the children from the day care center

ANS: C Personal health habits continue to contribute to the major causes of morbidity and mortality. The pattern of personal health habits and behavioral risk defines individual and family lifestyle risk. The client who doesn't move from the computer desk is creating great stress andstrain on personal physiology and needs to be educated on the benefits of exercise and the risks of cumulative trauma on the body. Multiple health benefits of regular physical activity have been identified; regular physical exercise is effective in promoting and maintaining health and preventing disease

Using the Neuman Systems Model, which of the following questions would the nurse ask a client to assess physiological health? a."What helps you to cope with situations involving your wife's cancer?" b."How has your child's illness affected the behavior of your other children?" c."Tell me about any illnesses your other family members have." d."Who do you turn to for support outside your immediate family?

ANS: C Physiological health involves issues related to physical wellness or illness. Other components of health in Neuman's model include psychological health, sociocultural health, developmental health, and spiritual health.

A nurse is providing education to a client about the use of PrEP. Which of the following statements would the nurse include as part of this teaching? a."Side effects of PrEP include extreme lethargy and joint pain." b."PrEP has been shown to be effective in preventing transmission of the disease from sharing needles." c."The effectiveness of PrEP will depend on your adherence to the medication regimen." d."PrEP will prevent you from contracting HIV and Hepatitis B."

ANS: C Pre-exposure prophylaxis, or PrEP, is a new HIV prevention method for people who do not have the infection but would like to reduce their risk of becoming infected. PrEP requires taking a pill to prevent the HIV virus from getting into the body. It has been shown to be effective for people at very high risk for HIV infection through sex; the results about its effectiveness with injection drug users are not yet available. This prevention method requires strict adherence to taking the medication and having regular HIV testing; it is also used in combination with other HIV prevention methods rather than in isolation (CDC: PrEP, 2012b)

A nurse is providing pre-conceptual counseling to a young woman. Which of the following supplements would the nurse most likely recommend? a. Iron b. Calcium c. Folic acid d. Vitamin C

ANS: C Research has shown that intake of folic acid can significantly reduce the occurrence of serious and often fatal neural tube defects by 50% to 70%. A recommendation was made that women capable of or planning a pregnancy take 0.4 to 0.8 mg of folic acid daily (USPSTF, 2009).

A nurse is implementing risk reduction interventions with a family. Which of the following questions is most important for the nurse to ask? a."Did any of the hunters in your family kill a deer this year?" b."How do you keep your rifles safe from curious children?" c."Where do you shoot with your handguns?" d."Where do you keep your rifles locked when it is not hunting season?"

ANS: C Risk reduction is a complex process that requires knowledge of risks and families' perceptionsof the nature of the risk. In this situation the nurse was asking questions to determine the family's perception of risks associated with owning guns. If the family does not perceive the behavior (having guns in the house) as risky, but rather as necessary for food or sport, the nurse must first educate or persuade the family that others may be more comfortable if certain precautionary measures are taken (such as locks on the guns). Rifles are used for food and sport but handguns are often used in crimes and accidents resulting in death

A nurse is implementing a tertiary prevention program to promote health among middle-aged women with diabetes. Which of the following best describes the intervention being implemented by the nurse? a. Developing lifestyle improvement programs for women at risk for diabetes b. Presenting lifestyle management presentations at women's conferences c. Monitoring blood glucose levels closely and modifying diet accordingly d. Screening glucose levels of women at risk for development of diabetes

ANS: C Tertiary prevention includes activities that are aimed to reduce the complications of the disease process. Only monitoring blood glucose levels is directed toward preventing problems in women who already are diagnosed with diabetes.

The nurse teaches the family of an AIDS client about managing symptomatic illness by preventing deteriorating conditions, such as diarrhea, skin breakdown, and inadequate nutrition. Which of the following best describes the action taken by the nurse? a. Primary prevention b. Secondary prevention c. Tertiary prevention d. Primary health care

ANS: C Tertiary prevention includes those interventions aimed at disability limitation and rehabilitation from disease, injury, or disability. Primary prevention refers to those interventions aimed at preventing the occurrence of disease, injury, or disability. Secondary prevention focuses on early detection and prompt treatment of disease, injury, or disability. Primary health care refers to the first line of care provided to patients typically by a physician or other health care provider.

A nurse advises a client who has HIV not to donate blood, plasma, or organs. Which of the following levels of prevention is being used? a. Primary prevention b. Secondary prevention c. Tertiary prevention d. Health promotion

ANS: C Tertiary prevention includes those interventions aimed at disability limitations and rehabilitation from disease, injury, or disability.

A nurse was involved in the initial implementation of the Healthy Communities and Cities initiative in the United States. Which of the following problems would have most likely been addressed? a. Reduction to the amount of pollutants released into the environment b. Provision of fire and police protection c. Creation of a coalition to address health disparities d. Elimination of crime and violence from the community

ANS: C The concept of a healthy community or city is based on the belief that the health of the community is largely influenced by the environment in which people live and that health problems have multiple causes: social, economic, political, environmental, and behavioral. Creation of a coalition engages residents in addressing the problem, which is impacted by multiple factors. The initial activities of this initiative focused on the problems of diverse populations. Reduction of the amount of pollutants and provision of fire and police protection do not address problems of diverse populations. Although crime and violence in a community may be decreased, it is not possible to eliminate it. REF: p. 443

A nurse arrives at a home at the appointment time established with the client over the phone. However, no one answers the door. Finally a child comes out and says, "My mom said she couldn't see you and you should go away." Which of the following actions should be taken by the nurse? a.Demand the child let the nurse into the home to talk to the mother. b.Interview the child as to how the family is doing. c.Leave a card with information on how to get in touch with the nurse . d.Point out that legally once an appointment has been made the mother needs to be seen

ANS: C The contact may be terminated as requested if the nurse determines that either the situation has been resolved or services have been obtained from another source and if the family understands that services are available and how to contact the agency if desired. However, the nurse should leave open the possibility of future contact. Obviously, the nurse cannot force entrance into the home. It would not be appropriate to coerce a child with misinformation or to interview a child about health concerns without a parent being present

A nurse organizes care for a family by focusing on the common tasks of family life and considering a longitudinal view of the family life cycle. Which theory is being applied? a.Family systems b.Structural-functional c.Family developmental d.Interactionist

ANS: C The family developmental theory focuses on common tasks of family life and provides a longitudinal view of the family life cycle

A case manager supports a client's decision to return home after having a total hip replacement rather than go to a skilled nursing facility. Which of the following phases of the nursing process is being used? a. Assessment b. Planning/outcome c. Implementation d. Evaluation

ANS: C The implementation phase is similar to the advocacy process when the nurse aids the client in decision making and supports the client's decisions. The assessment/diagnosis phase is similar to the advocacy process of information exchange, gathering data, and illuminating values. The planning/outcome phase is similar to the advocacy process when generating alternatives and consequences and prioritizing actions. The evaluation phase is most similar to the advocacy processes of affirmation, evaluation, and reformulation. REF: p. 492 (Table 22-2)

A nurse is working with an individual who pursues neither his concerns nor another's concerns. Which of the following conflict management behaviors is being used? a. Accommodating b. Collaborating c. Avoiding d. Compromising

ANS: C The individual would be "avoiding." Avoiding is defined as when an individual pursues neither his or her concerns nor another's concerns. Accommodating occurs when an individual neglects personal concerns to satisfy the concerns of another. Collaborating is when an individual attempts to work with others toward solutions that satisfy the goals of both parties. Compromising occurs when an individual attempts to find a mutually acceptable solution that partially satisfies both parties. REF: p. 494 (Box 22-3)

In the United States, which demographic group has the highest risk for HIV infection? a.African-American homosexual men b.Hispanic IV drug abusers c.White homosexual men d.White lesbian

ANS: C The largest number of new HIV infections in 2009 (11,400) were in white men who had sex with other men (MSM), and this was followed by black MSM (10,800)

The most chronic blood-borne infection in the United States is: a. hepatitis A. b. hepatitis B. c. hepatitis C. d. HIV.

ANS: C The most chronic blood-borne infection in the United States is hepatitis C. Hepatitis A virus is most commonly transmitted through the fecal-oral route. Sources may be water, food, feces, or sexual contact. The hepatitis B virus (HBV) is spread through blood and body fluids and, like HIV, is a blood-borne pathogen. HIV is transmitted through exposure to blood, semen, transplanted organs, vaginal secretions, and breast milk. Although hepatitis B and HIV are both blood-borne pathogens neither are the most chronic infection like hepatitis C.

A nurse is caring for a client who is HIV positive. Which of the following roles of the nurse would be the most important for the nurse to implement? a. Advocate, lobbying for AIDS research b. Counselor, discussing implications of future sexual activity c. Role model, providing supportive care d. Policy maker, addressing laws governing privacy rights of HIV-positive persons

ANS: C The priority role of the nurse should be role model, as nurses must be role models because many HIV-positive patients are stigmatized. The need to be an advocate and policy maker does not address the individual needs of the client. The nurse is able to counsel the client while providing supportive care.

A nurse is implementing the steps of the Community Health Promotion Model. Which of the following does this most resemble? a. Collaboration process b. WHO's Ottawa Charter c. Nursing process d. CDC's monitoring program

ANS: C The steps of the Community Health Promotion Model are very similar to the nursing process. The steps include assessment, planning, implementation, and evaluation. The collaboration process, WHO's Ottawa Charter, and the CDC's monitoring program do not contain steps similar to the Community Health Promotion Model. REF: p. 450

Which of the following situations would most likely indicate elder abuse? a. A daughter refuses to visit her mother due to work commitments. b. A child runs around a grandparent's house breaking items. c. A young man repeatedly steals money from his grandmother. d. An elderly person demands that the family come for dinner.

ANS: C Theft or mismanagement of money or resources is an element of abuse.

Which statement about eating disorders is correct? a. Individuals with anorexia frequently complain about weight loss. b. Purging is associated with anorexia. c. Most women with bulimia are concerned with the shape and weight of their body. d. Bulimia is considered to have more medical complications than anorexia.

ANS: C Those with bulimia are usually concerned with the shape and weight of their body. Those with anorexia view themselves as normal or overweight, purging is associated with bulimia, and anorexia is considered to have more complications than bulimia.

A nurse is caring for a client who has been diagnosed with a bacterial sexually transmitted infection (STI). Which of the following infections does this client most likely have? a. Trichomonas b. HIV c. Syphilis d. Genital warts

ANS: C Trichomonas, HIV, and genital warts are caused by viruses. Syphilis is caused by bacteria.

Which of the following provides the best explanation as to why people do not immediately seek medical treatment when they first become ill with HIV? a.They are afraid to get tested for fear results will be positive. b.They avoid the problem (maybe it will go away). c.They don't recognize their symptoms as possibly being due to HIV. d.It is too expensive to get an HIV test

ANS: C When HIV enters the body, it can cause a flu-like syndrome referred to as a primary infection or acute retroviral syndrome. This may go unrecognized. The symptoms are similar to flu or a bad cold including sore throat, lethargy, rash, fever, and muscle pain. An antibody test at this stage is usually negative. So the person or a medical provider may not recognize the illness as HIV

A nurse asks a family member, "What has changed between you and your spouse since your child's head injury?" Which of the following focuses of the family is the nurse assessing? a.The context b.The client c.A system d.A component of society

ANS: C When the focus is on the family as a system, the family is viewed as an interactional system inwhich the whole is more than the sum of its parts. The approach simultaneously focuses on individual members and the family as a whole at the same time. The interactions between family members are the target for nursing interventions

A nurse is completing an initial home visit with a family. Which of the following actions should be taken first by the nurse? a.Assess the family and the home setting for both strengths and problems. b.Determine the family's expectations of a home visit. c.Establish rapport between the nurse and the family. d.Engage in extended social interaction as would be expected from any guest

ANS: C With the exception of social interaction, all the tasks listed are important. The initial home visit includes the nurse's self-identification and clarification of role, establishing rapport with the family, assessing the situation, and then determining the client's expectations. Although insome senses the nurse is a guest in the home, the nurse is not there for social purposes but to help the family with health concerns. However, without rapport between the nurse and the client, the nurse will be notably less effective at other tasks

A bachelor's prepared nurse is providing case management services. Which of the following activities would most likely be provided by this nurse? a. Working with community aggregates b. Working with systems of disease c. Working with individuals d. Working with outcomes management processes

ANS: C Working with individuals would be the role of this nurse. Working with community aggregates, systems of disease, and outcomes management processes are implemented by an advanced practice nurse (APN). REF: p. 480

15. Which of the following public health actions has been particularly instrumental in reducing childhood infectious diseases in the United States? A. Answering parents' questions about the safety and importance of vaccines today B. Educational campaigns to all health care providers about the importance of immunizations whenever a child is seen C. "No shots, no school" legislation, which legally requires children be immunized before school D. Offering all immunizations to all children free of any charge

ANS: C "No shots, no school" legislation, which legally requires children be immunized before school Vaccines are one of the most effective methods of preventing and controlling communicable diseases. Hopefully, all nurses answer questions, remind colleagues to think about immunizations whenever a child is seen, and encourage continuing free or low-cost immunization clinics. One of the most effective programs has been the "no shots, no school" legislation, which has resulted in the immunization of most children by the time they enter school. DIF: Cognitive Level: Understand (Comprehension) REF: pp. 488-489

4. A man loudly protests his increased property tax bill right after the public health department has made a plea for more funds. "Why," he asks, "should my tax dollars be used to pay for their children to be immunized?" Which of the following would be the best response by the nurse? A. "Immunizations are required by law, and if their parents can't afford it, you and I will have to pay for it." B. "It's just the right thing to do." C. "Only by making sure most kids are immunized can we stop epidemics that might hurt all of us." D. "We're a religious God-fearing community, and we take care of each other."

ANS: C "Only by making sure most kids are immunized can we stop epidemics that might hurt all of us." Herd immunity is the resistance of a group of people to invasion and spread of an infectious agent because a high proportion of individual members of a group are resistant to the infection. Higher immunization coverage will lead to greater herd immunity, which in turn will block the further spread of the disease. DIF: Cognitive Level: Apply (Application) REF: p. 480

17. An instructor is reviewing Salmonella infections with her class. Which of the following comments indicates that the student needs further review on how Salmonella is spread? A. "Certain pets and farm animals may be Salmonella carriers." B. "It is possible to transmit Salmonella by person-to-person contact." C. "Salmonella may be spread by spores that form once contaminated blood is exposed to the air." D. "Salmonella outbreaks are usually due to contaminated meat, poultry, and eggs."

ANS: C "Salmonella may be spread by spores that form once contaminated blood is exposed to the air." Meat, poultry, and eggs are the foods most often associated with salmonellosis outbreaks. Animals are the common reservoir for the various Salmonella serotypes, although infected humans may also fill this role. Animals are more likely to be chronic carriers. Reptiles such as iguanas have been implicated as Salmonella carriers, along with pet turtles, poultry, cattle, swine, rodents, dogs, and cats. Person-to-person transmission is an important consideration in daycare and institutional settings. Anthrax (not Salmonella) forms spores when infected blood is exposed to air. DIF: Cognitive Level: Apply (Application) REF: p. 493

7. Which of the following is the most accurate description of a migrant farmworker? A. A person who does farm work as the primary means of employment, although other work may be done when the seasonal work ends B. A person who immigrates to the United States to "follow the crops" in performing seasonal farm work C. A person who moves from place to place to earn money performing seasonal agricultural work D. A person who specializes in the development of rural land for the purpose of farming

ANS: C A person who moves from place to place to earn money performing seasonal agricultural work The Office of Migrant Health of the U.S. Public Health Service defines a migrant farmworker as a person "whose principal employment is in agriculture on a seasonal basis, who has been so employed within the last 24 months, and who establishes for the purpose of such employment a temporary abode." The emphasis is on moving (temporary abode), farm work, and seasonal basis. Not all migrant farmworkers are immigrants. DIF: Cognitive Level: Remember (Knowledge) REF: p. 396

2. Which of the following best describes a health professional shortage area (HPSA)? A. An area with inadequate health care facilities for residents B. An isolated area of underserved populations within an urban region C. A region with insufficient numbers of health care providers D. A rural region of the United States with a population density of less than 10,000

ANS: C A region with insufficient numbers of health care providers An HPSA is a geographical area that has insufficient numbers of health professionals according to criteria established by the federal government. Often rural areas have a physician, nurse practitioner, or nurse in community health who provides services to residents who live in several counties. DIF: Cognitive Level: Remember (Knowledge) REF: p. 393

9. A migrant farmworker brings his daughter into the clinic with severe heat stroke from being out in the sun. The nurse explains the danger signs and stresses staying cool and drinking lots of water. The man seems to know this already. Which of the following best describes the most likely reason that this happened? A. It was a rare occurrence, which probably won't be repeated. B. The daughter either disobeyed her father and went out to play in the sun or just did not realize how hot she was. C. Children may work on small farms because the family may need the additional income. D. The parents were busy working and didn't realize the child was outside so long.

ANS: C Children may work on small farms because the family may need the additional income. Children 12 to 13 years of age can work on a farm with the parents' consent or if the parent works on the same farm. Children younger than 12 years can work on a farm with fewer than 7 full-time workers. Children may need to work for the family's economic survival. DIF: Cognitive Level: Apply (Application) REF: pp. 399-400

9. Which of the following data would most likely be collected in a syndromic surveillance system? A. Incidence of bioterrorism attacks B. Number of air travelers C. Incidence of school absenteeism D. Number of influenza vaccines administered

ANS: C Incidence of school absenteeism Syndromic surveillance systems use existing health data in real time to provide immediate analysis and feedback to those charged with investigation and follow-up of potential outbreaks. These systems incorporate factors such as the previously mentioned temporal and geographic clustering and unusual age distributions with groups of disease symptoms or syndromes (e.g., flaccid paralysis, respiratory signs, skin rashes, gastrointestinal symptoms) with the goal of detecting early signs of diseases that could result from a bioterrorism-related attack. Syndromic surveillance systems may include tracking emergency department visits sorted by syndrome symptoms as well as other indicators of illness including school absenteeism and sales of selected over-the-counter medications. In recent years, the tracking of cold medicines used to make crystal methamphetamine has received considerable attention. DIF: Cognitive Level: Understand (Comprehension) REF: pp. 481-482

18. Which of the following is the most common vector-borne disease in the United States? A. Babesiosis B. Ehrlichiosis C. Lyme disease D. Rocky Mountain spotted fever

ANS: C Lyme disease All four are diseases borne by ticks as the vectors. Lyme disease became a nationally notifiable disease in 1991 and is now the most common vector-borne disease in the United States. DIF: Cognitive Level: Understand (Comprehension) REF: p. 494

10. A community is experiencing an epidemic of the measles. The nurse is trying to determine if this problem is happening in other communities as well. Which of the following resources should the nurse use to answer this question? A. Centers for Disease Control and Prevention Weekly Report B. Communicable Diseases Weekly Report C. Morbidity and Mortality Weekly Report D. Weekly National Report of Communicable Diseases

ANS: C Morbidity and Mortality Weekly Report Requirements for disease reporting in the United States are mandated by state rather than federal law. The list of reportable diseases varies by state. State health departments, on a voluntary basis, report cases of selected diseases to the Centers for Disease Control and Prevention (CDC) in Atlanta, Georgia. The National Notifiable Diseases Surveillance System (NNDSS) data are collated and published weekly in the Morbidity and Mortality Weekly Report (MMWR). DIF: Cognitive Level: Apply (Application) REF: p. 482

5. A migrant farmworker presents to the clinic reporting an acute onset of severe abdominal pain, nausea, vomiting, diarrhea, and headache with difficulty concentrating. Which of the following conditions would cause such symptoms? A. Appendicitis B. Bacterial gastroenteritis C. Pesticide poisoning D. Viral illness

ANS: C Pesticide poisoning Although all of these conditions can cause some or all of the symptoms listed, because a migrant farmworker is employed in agriculture, the most probable cause is pesticide poisoning. Acute health effects of pesticide exposure include mild psychological and behavioral deficits such as memory loss, difficulty with concentration, and mood changes, abdominal pain, nausea, vomiting, diarrhea, headache, malaise, skin rashes, and eye irritation. DIF: Cognitive Level: Analyze (Analysis) REF: p. 395

25. Which of the following symptoms suggests smallpox as opposed to the more common and much less dangerous chickenpox? (Select all that apply.) A. Child appears only mildly ill until late stages in smallpox. B. Lesions appear in various stages in the same area of the body rather than all at once. C. Rash lesions are most abundant on the face and extremities, not on the trunk. D. Rash occurs 2 to 4 days after sudden onset of fever rather than with the fever.

ANS: C, D C. Rash lesions are most abundant on the face and extremities, not on the trunk. D. Rash occurs 2 to 4 days after sudden onset of fever rather than with the fever. Symptoms of smallpox include rash lesions on face and extremities, rash that occurs 2 to 4 days after onset of fever, and vesicles that are deep seated. DIF: Cognitive Level: Understand (Comprehension) REF: p. 488

In comparison with traditional norms, which family functions have become increasingly important in modern American society? (Select all that apply.) a.Conferring appropriate social status b.Educating the younger members c.Ensuring physical and mental health d.Fostering interpersonal relationships and support

ANS: C, D Historically, families have had several functions including financial survival, reproduction, protection from hostile forces, and enculturation, including religious faith, education, conferring social status. Today, however, the more important functions are fostering relationships (emphasizing how people get along and their level of satisfaction) and promoting physical and mental health

The nurse and the family have agreed on an ambitious goal to improve family functioning, but as the family later expresses with some dismay, they have not been able to change their behavior as easily and quickly as they had hoped. Which of the following must be remembered throughout this process? (Select all that apply.) a.A reassessment of resources should be done if the plan does not work. b.Individual family members must all be willing to make the plan their first priority. c.Goals must be realistic and feasible. d.Ongoing negotiation is central to the process

ANS: C, D In contracting, an important aspect is obtaining the family's view of the situation and its needsand problems. Goals must be mutually set and realistic. A pitfall for nurses and clients who are new to contracting is to set overly ambitious goals. Because contracting is a process characterized by ongoing renegotiating, the goals are not static. The family's inability to change "as easily and quickly as they had hoped" does not mean the plan is not workable—only that more time and effort may be necessary

A health care provider is working with elderly clients who have ongoing chronic disease. Which of the following strategies can best assist them with healing? (Select all that apply.) a. Advocating for increased support for elderly persons b. Eliminating signs and symptoms of disease c. Managing any chronic diseases to prevent complications and delay deterioration d. Maximizing self-care capacity

ANS: C, D With chronic illness, the focus is on healing (a unique process resulting in a shift in the body/mind/spirit system) rather than curing (elimination of the signs and symptoms of disease). Appropriate goals include maximizing self-care capacity, managing chronic diseases effectively, preventing complications, delaying deterioration and decline, and achieving the highest possible quality of life before dying with comfort, peace, and dignity.

A nurse is implementing interventions to improve the health of the community. Which of the following approaches should be taken by the nurse? a. A top-down approach with rational-empirical problem solving b. A top-down approach with community practice planned by experts c. A bottom-up approach with facilitation of communication d. A bottom-up approach with multisectoral planning and action for health

ANS: D A bottom-up approach with multisectoral planning and action for health incorporates community participation. A bottom-up approach uses broad-based community problem solving that includes health professionals, local officials, service providers, and other community members, including those at risk for health problems. The top-down approach describes when professionals and experts tell the citizens what to do rather than involving and asking them. Social planning stresses rational-empirical problem solving, usually by outside professional experts with a top-down approach. This approach does not focus on building community capacity or fostering fundamental social change. REF: p. 444

Which of the following behaviors results in men being less healthy than women? a. Concentration on sports, hunting, and other dangerous recreational choices b. Employment in stressful positions more so than women c. Preference to spend money on priorities other than health care d. Reluctance to visit physicians

ANS: D A major obstacle to improving men's health is their apparent reluctance to consult their primary care provider. Men are not well connected to the health care system. Men do not participate in health care at the same level as women, apparently because of the traditional masculine gender role learned through socialization (Bonhomme, 2007). Only 57% of U.S. men see a doctor, nurse practitioner, or physician assistant compared with 74% of women (AHRQ, 2010).

A nurse is performing a health assessment on a young adult female. Which of the following comments by the client would cause the nurse to be concerned? a. "I can't believe I didn't get that promotion. I thought I was the best candidate." b. "I can't believe they didn't admit me to that graduate program with my good undergraduate grades!" c. "I can't believe how lonely it is around the house since my spouse died." d. "I can't believe how little I care about anything anymore since that diagnosis."

ANS: D A number of factors contribute to depression, including being female, having a family history of depression, unemployment, and chronic disease. All the options represent a loss, either potential or actual, but the statement about not caring about anything represents an actual symptom of depression.

A student asks the nurse at the student health clinic how AIDS is diagnosed. Which of the following statements would be the best response by the nurse?a."A diagnosis of AIDS is made when a screening test called an enzyme-linked immunosorbent assay (ELISA) is confirmed by the Western blot test." b."A diagnosis of AIDS is made when antibodies to HIV are detected about 6 weeksto 3 months following possible exposure." c."A diagnosis of AIDS is made when antibodies to HIV reach peak levels of 1000/ml of blood." d."A diagnosis of AIDS is made when CD4 T lymphocytes drop to less than 200/ml."

ANS: D AIDS is defined as a disabling or life-threatening illness caused by HIV; it is diagnosed in a person with a CD4 T-lymphocyte count of less than 200/ml with or without documented HIV infection. The HIV antibody test (usually the EIA) is the most commonly used screening test for determining whether the antibody to HIV is present but does not confirm AIDS. Positive results with the EIA are tested further with the Western blot test. However, false-negative results are frequent between 6 weeks and 3 months following exposure

Which of the following statements best explains why many health care providers are more afraid of getting hepatitis B than HIV? a.Everyone would assume the person infected with hepatitis B is a drug user. b.Having HBV would mean no further employment in health care. c.The fatality rate is higher and occurs sooner with HBV. d.There is no treatment for HBV, which can be a very serious illness

ANS: D Both HBV and HIV are blood-borne pathogens. Health care workers may be exposed to eitherfrom needle stick injuries and mucous membrane splashes. However, HBV remains alive outside the body for a longer time than does HIV and thus has greater infectivity. The virus can survive for at least 1 week dried at room temperature on environmental surfaces, and therefore infection control measures are crucial in preventing transmission. There is no treatment for hepatitis B, and although some persons never have symptoms and others fight off the disease, many suffer from chronic hepatitis B, a very serious illness. Others may become chronic carriers of the disease

A nurse is working as a case manager. Which of the following best describes the diagnoses that the case manager is most likely to encounter? a. Bankruptcy, financial distress, and depression b. Flu, colds, and frequent headaches c. Malaria, bird flu, and Dengue fever d. AIDS, spinal cord injury, and ventilator dependency

ANS: D Case-managed conditions include many chronic conditions. AIDS, spinal cord injury, and ventilator dependency are the only chronic conditions listed. The other options are acute or nonmedical conditions REF: p. 479

A nurse wants to empower the family of a mother who has been newly diagnosed with breast cancer. Which of the following actions would the nurse most likely take?a.Apply for emergency financial assistance on the family's behalf. b.Arrange for community members to assist with child care. c.Invite the mother to join a cancer support group. d.Teach the family how to navigate the health care system

ANS: D Definitions of empowerment reflect three characteristics of the empowered family seeking help: access and control over needed resources, decision-making and problem-solving abilities, and the ability to communicate and to obtain needed resources. Approaches for helping individuals and families assume an active role in their health care should focus on empowering, rather than giving direct help

A nurse is assisting clients to improve their health status. Which of the following types of management is being used by the nurse? a. Care management b. Case management c. Disease management d. Demand management

ANS: D Demand management seeks to control use by providing clients with correct information and education strategies to make healthy choices, to use healthy and health-seeking behaviors to improve their health status, and to make fewer demands on the health care system. Care management is an enduring process in which a manager establishes systems and monitors the health status, resources, and outcomes for a targeted aggregate of the population. Case management is defined as a collaborative process of assessment, planning, facilitation, care coordination, evaluation, and advocacy for options and services to facilitate an individual's and family's comprehensive health needs through communication and available resources to promote quality cost-effective outcomes. Disease management constitutes systematic activities to coordinate health care interventions and communications for populations with disease conditions in which client self-care efforts are significant. REF: p. 479

A nurse was reading PPD tests 24 hours after another nurse had administered them. Which of the following findings would cause the nurse to interpret the test as positive? a.15 mm of erythema in a client with HIV infection b.5 mm of induration in an immigrant from a country where TB is endemic c.A 5-mm ruptured pustule with purulent drainage in a homeless client d.10 mm of swelling and increased firmness in a client recently released from a correctional facility

ANS: D Erythema alone does not indicate a positive finding. For a PPD test to be positive, induration (swelling with increased firmness) must be present. A diameter of 10 mm induration would bea positive finding in an immigrant from a region with high TB infection. A small pustule in a homeless client undoubtedly is an infection but may not be due to the PPD test

A nurse is conducting a family assessment. Which of the following behaviors would the nurserecognize as suggestive of a family with problems? a.Before eating, the family prayed, expressing gratitude for their blessings. b.During family play, jokes and laughter were heard. c.Each person had a private room with a door for alone time. d.Most of the conversation was between the father and the eldest daughter

ANS: D Evidence of healthy families can be seen in a variety of observations, including open communication among all members, mutual play with humor, balanced interactions among allmembers, expressions of a religious core or other value system, and each member being allowed some privacy

Which of the following terms refers to government actions that have a direct or indirect effect on families?a.Family funding b.Family legislation c.Family planning d.Family policy

ANS: D Government actions that have a direct or indirect effect on families are called family policy

The correctional health nurse is doing a quick assessment on a newly admitted inmate who is HIV positive. Which of the following diseases should the inmate receive screening for immediately? a.Herpes zoster b.Hepatitis B c.Hepatitis C d.Tuberculosis

ANS: D HIV-infected persons who live near one another, such as in correctional facilities, must be carefully screened and deemed noninfectious before admission to such settings. A person withHIV is more susceptible to opportunistic infections, the most common of which is TB

A nurse is conducting a health risk appraisal. Which of the following activities is the nurse assessing when using this tool? a.Health promotion activities b.Illness prevention activities c.Risk reduction activities d.Unhealthy activities

ANS: D Health risk appraisal refers to the process of assessing for the presence of specific factors in each of the categories that have been identified as being associated with an increased likelihood of an illness, such as cancer, or an unhealthy event, such as an automobile accident.Therefore, the nurse would assess for unhealthy behavior and activities in the areas of biological and age-related risk, social and physical environment risk, and behavioral risk. Activities to promote health, prevent illness, or reduce risks would be advantageous, not risky

An elderly person is in the last stages of dying. Which type of care would be the best for him? a. Home health b. Assisted living c. Nursing home d. Hospice

ANS: D Hospice would be the best form of care for a person in the last stages of dying.

A nurse is working in a public health center. A patient who has been newly diagnosed as HIV positive comes for counseling. By law, which of the following actions must be taken by the nurse? a. Give antiviral medications to the patient b. Ask the person to name all of his or her sexual contacts c. Refer the patient to the social worker as someone possibly needing case management d. Report the HIV-infected person to the state health department

ANS: D It is mandatory to report HIV cases by name in all states, although not all states require viral load and CD4 counts. There are not laws in place that mandate administration of medications, naming of sexual contacts, or referral to case management.

Care in a managed system is provided by less experienced providers. Which of the following ethical principles is being influenced in this situation? a. Autonomy b. Beneficence c. Nonmaleficence d. Justice

ANS: D Justice calls for equal distribution of health care with reasonable quality. Autonomy is the individual's right to choose a provider. Beneficence is influenced when excessive attention to cost containment supersedes or impairs the nurse's duty to provide measures to improve health or relieve suffering. Nonmaleficence is doing no harm, which is addressed when incorporating outcomes measures, evidence-based practice, and monitoring processes in plans of care. REF: p. 497

Which of the following will provide the highest benefit to an older adult's well-being? a. Eligibility for Medicare and Social Security b. Higher socioeconomic status, income, and education c. Senior citizen privileges such as senior citizen centers and senior citizen discounts d. Social networks that give support and meaning to life

ANS: D Medicare, Social Security, senior privileges, and high socioeconomic status, income, and education are all extremely helpful to older adults. However, social networks that give support and meaning to life are a major factor that can contribute to ongoing health and vitality. One of the biggest dangers is social isolation.

The following people enter the health clinic together: an unmarried man and his year-old son, an unmarried woman with a year-old daughter, and the man's married brother, who is separated from his wife. During the assessment it is determined that both men work and contribute to the household, where all of them live. Which of the following best describes the "family"?a.The group consists of three families: the man and his son, the woman and her daughter, and the brother, who is married even though he and his wife are separated. b.There are two families involved: first, the unmarried man and woman and their two children, and second, the brother, who is married even though he and his wife are separated. c.There is no family here, only three adults sharing resources between themselves and two biologically related children. d.The family includes whoever the adults state are family members.

ANS: D Nurses working with families should ask an adult member to identify all those considered to be family members and then include those members in health care planning. A "family" may range from the traditional nuclear model with extended family to such "postmodern" family structures as single-parent families, stepfamilies, same-gender families, and families consisting of friends

The case rate of new TB is highest among which ethnicity in the United States? a. African American b. Native American c. White American d. Asian American

ANS: D Of the new cases, 59% are foreign-born persons living in the United States, with Asians and Hispanics being the most common ethnic groups, representing 30% and 28% of national TB cases. The TB rates for African American, Native American, and White American are lower than the Asian and Hispanic populations.

A nurse is implementing the Community Health Promotion Model. Which of the following underlying principles should the nurse use in order to facilitate active participation? a. Individuals should work in groups to complete necessary tasks. b. Individuals participating should represent all segments of the population. c. People will work together if they like the leadership. d. People will participate when they see the issues as worthy of their time.

ANS: D People participate when they feel a sense of community and see their involvement in the issues as relevant and worthy of their time. This is the second step of the Community Health Promotion Model. There should be a broad-based representation on the planning committee, but it may not represent all segments of the population. The tasks may need to be completed by both groups and individuals. Although the likability of leadership may influence who participates, it most likely will not influence whether they are able to work together. REF: p. 450

A home health nurse is about to visit a family at their home. However, the nurse is feeling uncomfortable about getting out of her car because a group of young adults across the street are drinking and fighting among themselves. Which of the following actions should be taken by the nurse? a.Call the agency and ask what she should do. b.Call the family, explain the situation, and try to reschedule. c.Fulfill the nurse's commitment to the family and enter the home quickly. d.Drive away and notify the family from a safer location

ANS: D Personal safety is an issue. Home visits are generally very safe; however, as with all worksites, the possibility of violence exists. Therefore, the nurse needs to use caution. If a reasonable question exists about the safety of making a visit, the nurse should not make the visit

A client is being treated for secondary syphilis. Which of the following signs and symptoms would the nurse anticipate the client would exhibit? a.Chancre at the site of entry b.Jaundice c.Difficulty coordinating muscle movements d.Skin rash without itching

ANS: D Secondary syphilis occurs when the organism enters the lymph system and spreads throughout the body. Signs include skin rash on one or more areas of the body and do not cause itching. Other symptoms may include fever, swollen lymph glands, sore throat, patchy hair loss, headaches, weight loss, muscle aches, and fatigue

A woman needs to take some time off from work to care for her invalid mother. Which of the following health policies allows her to take an extended leave from work to care for a family member? a. Personal Responsibility and Work Opportunity Reconciliation Act b. Women's Health Equity Act (WHEA) c. Temporary Assistance for Needy Families (TANF) d. Family and Medical Leave Act (FMLA)

ANS: D The FMLA provides job protection and continuous health benefits where applicable for eligible employees who need extended leave for their own illness or to care for a family member

A nurse is using the provisions of the Family Medical Leave legislation. Which of the following actions is the nurse most likely to take? a.Resigning from employment, but retaining health insurance b.Sharing family information with colleagues c.Providing Medicaid to a family who cannot afford health insurance d.Taking a defined time off of work for family events without fear of job loss

ANS: D The Family Medical Leave legislation allows for a family member to take a defined amount ofleave for family events, such as births and deaths, without fear of losing his or her job

A high school student is planning to volunteer at the hospital after school, so she needs to have a Mantoux test before beginning. Which of the following information should the nurse provide to the new volunteer? a."I will be using tiny tines to administer the TB antigen to the skin on your arm." b."Notify the clinic immediately if you experience any redness or itching at the test site." c."The areas should be kept dry until you return; cover it with plastic wrap when bathing." d."You will need to return in 2 to 3 days to have any reaction interpreted."

ANS: D The Mantoux test is a TB skin test that involves a 0.1-ml injection containing 5 tuberculin units of PPD tuberculin (not tines as in the TB tine test). The site should be examined for a reaction 48 to 72 hours (2 to 3 days) after injection. Only induration should be measured, and the results should be recorded in millimeters

A client who is very upset says to the nurse, "But we always used a condom! How could I have genital warts?" Which of the following would be the best response by the nurse? a."Are you positive you always used a condom?" b."Condoms don't always work." c."The condom might have had a tear in the latex." d."Skin to skin contact to a wart may have occurred outside the area that the condom covers."

ANS: D The challenge of HPV prevention is that condoms do not necessarily prevent infection. Warts may grow where barriers, such as condoms, do not cover, and skin-to-skin contact may occur

A client was clearly very relieved when an HIV test came back negative. "Thank goodness. I've had sex several times without a condom, and when one of my friends said he was sick, I think I panicked." Which of the following would be most important to emphasize to the client immediately? a.Abstinence is the only way to be certain you are HIV-free. b.Sex should be restricted to one partner. c.The test could be wrong and the client might still have an HIV infection. d.The test would not cover any recent infection, so if the client has had recent unprotected sex, the test should be repeated in 3 months

ANS: D The client needs to understand that the test cannot identify infections that may have been acquired within the previous 3 months before the test. Appearance of the HIV antibody can take up to 12 weeks. All persons who have an STD test should be counseled about risk-reduction activities before and after the test is done

When the Healthy Communities and Cities concept was brought to the United States: a. the cities chosen for the initial work were Los Angeles and New York City. b. Canada adopted the program at the same time for continuity across borders. c. the same strict guidelines that were implemented in Europe were used. d. smaller communities and localities were targeted instead of large cities.

ANS: D The concept of Healthy Communities was used to incorporate localities that were not necessarily cities. The initial projects were Healthy Cities Indiana and the California Healthy Cities project. Healthy Communities and Cities operates in other parts of the world under different names and national networks have been developed among countries. There are not strict guidelines for implementation of this program. REF: p. 443

A nurse is designing interventions and evaluating results of Healthy Municipalities and Cities. Which of the following considerations should be made by the nurse? a. Influence of individual actions b. Acute care hospital services in the community c. Social results and endorsement of law makers for policy changes d. Effectiveness of health services, including delivery of preventive services

ANS: D The effectiveness of health services, including the delivery of preventive services, should be considered when designing interventions and evaluating the results of Healthy Municipalities and Cities. Influence of individual actions, acute care hospital services, and social results and endorsement of law makers are not part of Healthy Municipalities and Cities. REF: p. 451 (Box 20-3)

A case manager has contacted providers and has negotiated services and prices. Which of the following phases of the nursing process is being demonstrated? a. Assessment b. Diagnosis c. Planning/outcome d. Implementation

ANS: D The implementation phase of the nursing process occurs when a case manager advocates for clients' interests and arranges for the delivery of service. Examples of assessment include developing networks with target populations and dissemination of written materials. Diagnosis includes the identification of a problem/opportunity. Examples of activities used during the diagnosis phase include holding conferences, determining conclusions on the basis of assessment, and using interprofessional teams. Examples of planning for outcomes include validating and prioritizing problems and selecting evidence-based interventions. REF: p. 282 (Table 22-1)

The nurse in community health needs to conduct a family assessment within a commune but is uncertain how to proceed because family lines appear blurred. The best way to determine the family of a mother and her child is to ask the woman which of the following questions? a."How many children do you have, and who is the father of each?" b."Is there a register of families who are members of this commune?" c."Tell me about your significant other." d."Who are the members of your and your child's family?"

ANS: D The members of a family are self-defined. The family includes whoever the woman says are family members. The nurse should include all those members in health care planning.

The most common reportable infectious disease in the United States is: a. gonorrhea. b. syphilis. c. herpes. d. chlamydia.

ANS: D The most common reportable infectious disease in the United States is chlamydia. Gonorrhea is the second most commonly reported infectious disease. Syphilis and herpes are not among the most common reportable infectious diseases in the United States.

A case manager is fulfilling the role of coordinator. Which of the following nursing interventions would most likely be completed? a. Providing information to all parties about the situations affecting the client b. Educating the client and providers in order to make informed decisions c. Supporting all parties to work toward mutual goals d. Arranging, regulating, and balancing needed health services for the client

ANS: D The role of coordinator is fulfilled when the nurse arranges, regulates, and balances needed health services for the client. Providing information to all parties about the client's situation is the role of the monitor/reporter. Educating the client and provider in order to make informed decisions is the role of educator, and supporting all parties to work toward mutual goals is the role of facilitator. REF: p. 484 (Box 22-1)

Which of the following theories views the family as a whole with boundaries that are affected by the environment? a.Family developmental theory b.Structural-functional theory c.Family role theory d.Family systems theory

ANS: D The theory that views the family as a whole with boundaries that are affected by the environment is the family systems theory.

Which of the following groups would be most likely to receive an injection of prophylactic immune globulin for possible exposure to hepatitis A? a. Persons who have had direct contact with blood b. Those who ate at the same restaurant as the person with hepatitis A c. All health care workers d. All those who had household or sexual contact with persons with hepatitis A

ANS: D Those who have been in close contact with persons who develop hepatitis A should receive immune globulin. The other groups do not describe populations in direct contact with the person who has hepatitis A.

11. There is great concern in the nurse's community over three local cases of West Nile virus. Which of the following actions should the nurse take to get the community involved in addressing this problem? A. Ask the state department of health for assistance B. Demand that everyone over age 65 become immunized immediately C. Encourage immunization of all children under 12 D. Have an educational campaign to remove any containers of standing water

ANS: D Have an educational campaign to remove any containers of standing water Periodic outbreaks of West Nile virus appear to result from a complex interaction of multiple factors, including weather—especially hot, dry summers followed by rain, which influences mosquito breeding sites and population growth. Removing standing water will remove mosquito breeding sites. DIF: Cognitive Level: Apply (Application) REF: pp. 483-484

23. Which of the following infectious disease interventions best represents the use of secondary prevention? A. Clients with HIV infection are encouraged to use condoms to protect sexual partners. B. Clients with HIV infection are given medications to improve immunological response. C. Health care workers are encouraged to receive annual vaccinations for influenza. D. Health care workers are required to have a tuberculosis skin test or chest x-ray.

ANS: D Health care workers are required to have a tuberculosis skin test or chest x-ray. Tuberculosis screening for health care workers is an example of secondary prevention. TB skin tests and chest x-rays are methods of tuberculosis screening. Encouraging clients with HIV to use condoms is an example of primary prevention, because the goal is to prevent exposure to the partner. Encouraging annual influenza vaccinations is also an example of primary prevention. Giving clients with HIV medications is an example of tertiary prevention, because the goal is to reduce complications in those already having the infection. DIF: Cognitive Level: Apply (Application) REF: p. 487

3. During an outbreak of hepatitis A, nurses are giving injections of hepatitis A immunoglobulin to selected susceptible persons. Which of the following best describes the type of immunity that will follow the administration of these injections? A. Active immunity B. Long-lasting immunity C. Natural immunity D. Passive immunity

ANS: D Passive immunity Passive immunity refers to immunization through the transfer of a specific antibody from an immunized individual to a nonimmunized individual, such as the transfer of antibody by administration of an antibody-containing preparation (immune globulin or antiserum). Passive immunity from immune globulin is almost immediate but short-lived. It often is induced as a stopgap measure until active immunity has had time to develop after vaccination. DIF: Cognitive Level: Apply (Application) REF: p. 480

21. At a town meeting with public health officials to discuss a communicable disease outbreak, a nurse is asked to explain what is meant by the phrase "a virulent organism." The nurse explains that this means the organism causing the disease is able to do which of the following? A. Bypass normal immunological response mechanisms B. Invade major organ systems C. Produce toxins and poisons that weaken the body D. Produce very severe physical reactions

ANS: D Produce very severe physical reactions Virulence is the ability to produce a severe pathological reaction. DIF: Cognitive Level: Apply (Application) REF: p. 480

8. A student engages in unprotected sex under the influence of alcohol. The student decides to have an HIV test completed the next day. Which of the following results will most likely occur? A. The results will probably be negative for HIV. B. The results will probably be positive for HIV. C. The probability of disease is so low there is no reason to be tested. D. The test results won't be reliable so soon after exposure.

ANS: D The test results won't be reliable so soon after exposure. It may take up to 6 months after exposure to the HIV virus before an HIV antibody test can test positive, although most infected people will test positive within 3 months. A negative test, therefore, is not a reliable indicator of infection status if exposure is very recent. The incubation period or the time interval between invasion by an infectious agent and the first appearance of signs and symptoms of the disease may be between 10 and 15 years for AIDS. DIF: Cognitive Level: Analyze (Analysis) REF: p. 481

8. An employer provides a migrant farm family the day off to visit the health clinic in a nearby community and tells them to take all of the time they need. However, the family arrives at the clinic appearing very stressed. In addition to the health issue, which of the following would most likely be a fear experienced by the family? A. Their personal belongings may be stolen while they are at the clinic. B. Immigration officials will send them back to their home country. C. The clinic personnel will look down on them and be biased against them. D. They weren't getting paid for that day, and continued employment is never certain.

ANS: D They weren't getting paid for that day, and continued employment is never certain. Any of the answers might be true; however, most migrant farmworkers are legal residents or U.S. citizens, not illegal immigrants. Many do not have their own transportation and must rely on friends. Migrant farmworkers often have an unpredictable and difficult lifestyle. Many must leave home each year and travel to distant locations to work. They may be uncertain about their work and housing as they go from one possible job to another. An employer telling them to "take all the time they want" could be interpreted as meaning they may no longer be needed, and their next job may be very uncertain. DIF: Cognitive Level: Analyze (Analysis) REF: p. 398

10. A nurse is working with Mexican immigrants. Which of the following behaviors would most likely lead to a positive interaction for the nurse? a. Avoiding touching the client except when necessary as part of the physical examination b. Calling the client by name, socializing before addressing the problem, and being very respectful c. Keeping all interactions direct, to the point, and targeted on the reason for presentation d. Maintaining a non-confrontational relationship by avoiding any disagreement even if the nurse does disagree with what the client is saying

B

12. Which of the following conditions of the rural environment provides increased opportunities for teaching? a. Increased interaction among residents due to neighbors visiting neighbors on the family farms b. Involvement in rural community activities provides more contact with community residents than in urban areas c. Nursing responsibilities in these areas stress the importance of primary, secondary, and tertiary prevention d. Increased illnesses and injuries of rural residents require that they see nurses more often

B

A family nurse is working with a married couple that has decided to remain child-free. The nurse recognizes this decision as a: A. biological necessity. B. contemporary family function. C. religious belief decision. D. threat to family survival.

B

A nurse in community health is working with a single parent of three children, ages 4, 6, and 8. The 6-year-old has cerebral palsy. The 4-year-old has asthma. The maternal grandmother lives with the family and has diabetes. The nurse understands the importance of working within the context of the existing family structure and community resources because families are: A. resistive to outside intervention or involvement. B. responsible for providing/managing the care of their members. C. unable to manage the stress of complex health needs. D. restricted in their ability to identify interventions.

B

A nurse is in the termination phase of the nurse-family relationship. Which of the following strategies would the nurse most likely implement? a. Increasing sessions with the nurse b. Making referrals when appropriate c. Providing a formative evaluation of the relationship d. Refusing additional communication with the family

B

A nurse is making an appointment with a family for a nursing visit. Which of the following describes a potential barrier the nurse may encounter? a. The assessment cannot be done unless the extended family is present. b. It may be difficult to find a convenient time for all family members to be present. c. Nurses have limited time to do home visits. d. Families are often scattered over a large area, making access difficult.

B

A nurse is working with a family who is confronting major challenges to their health. Which of the following approaches would be most helpful for the nurse to use? a. Allowing the family to be noncompliant b. Building on the family's strengths and resilience c. Labeling the family as resistant d. Recognizing that the family is dysfunctional

B

The hospital-based nurse has worked with a client at some length regarding appropriate diet. Based on the family systems theory, which of the following will most likely occur when the client returns home? a. The family member who prepares food will probably suggest the newly discharged member eat the meals everyone in the family enjoys. b. The family member who prepares food will probably try to modify family meals without obvious change for the family as a whole. c. The family member who prepares food will probably prepare meals based on the diet plan for all the family. d. The family member who prepares food will probably prepare special meals for the newly discharged member.

B

The nurse is told that a healthy, functional family consisting of a 25-year-old man and a 24-year-old woman, who are expecting their first child, would appreciate a nurse coming to their apartment for anticipatory guidance in preparing themselves and their apartment for the baby. Based on that statement, which of the following assumptions can the nurse safely make about the family? a. The family is lacking a strong support system. b. The family's basic needs are being met. c. The couple's in-laws are unavailable to share their expertise about child care. d. The married couple is excited about their first baby.

B

Which of the following factors must be considered before deciding on an appropriate plan of action? a. Family agrees to the nurse's plan. b. Family is capable of the required actions. c. Family will learn better coping skills from the nurse's plan. d. Nurse has informed family how to complete the required actions.

B

To reach the desired goal of maximizing the full immunization rates for preventable communicable disease and increasing herd immunity levels, it is crucial for the nurse to: a) assume that the primary care physician has provided all appropriate immunizations b) check an individual's immunization status at each and every visit c) support parents who are reluctant to immunize their children d) understand the difficulties in obtaining and maintaining immunization schedules

B Diseases such as polio, diphtheria, pertussis, and measles, which previously occurred in epidemic proportions, are now controlled by routine childhood immunization, but they have not been eradicated. Despite the availability of free vaccines, many infants and toddlers, who are the most vulnerable to these potentially severe diseases, are not receiving scheduled immunizations by their second year. Recent "no shots, no school" legislation has increased the number of children fully immunized by the time they enter school. Increasing the rate of full immunization can provide the general public the protection of herd immunity, which is helpful in controlling communicable disease exposure through immigration, endemic (pertussis), and epidemic (influenza) outbreaks.

A student complains to the college health nurse that her academic work has been going downhill because of lack of sleep. "My 3-year-old probably misses her babysitter since she has started going to the big daycare center. She hasn't been sleeping well and keeps scratching her bottom. Hopefully, she'll adapt to daycare soon." Which of the following information should the nurse provide to the student? a) "Dry skin in winter weather can cause itchiness; try to put on lotion before bedtime." b) "Your daughter may have pinworms; let me teach you how to check for this." c) "Perhaps your child is not developmentally ready for group play." d) "Try to arrange more one-on-one time with your 3-year-old."

B Enterobiasis (pinworm infection) is the most common helminthic infection in the United States with about 42 million cases a year. This infection is seen most often among children in institutional settings. Pinworms cause itching, especially around the anus, which can result in a lack of sleep for both child and caregiver.

Which of the following is the most common vector-borne disease worldwide? a) Dengue b) Malaria c) Onchocerciasis (river blindness) d) Yellow fever

B Globally, malaria is the most prevalent vector-borne disease, with over 2.4 billion people at risk and more than 275 million cases reported each year. More than 1 million children die of malaria each year. Dengue is the second most common vector-borne disease.

A student comes to the college health clinic with typical cold symptoms of fever, sneezing, and coughing, but the nurse also notes small white spots on the inside of the student's cheeks. Which of the following actions should be taken by the college health nurse? a) Inform all students, staff, and faculty of a possible rubella epidemic b) Inform all students, staff, and faculty of a possible measles epidemic c) Reassure the student that it is just a bad cold and will soon pass d) Tell the student to take two acetaminophen and drink lots of fluids

B Measles is an acute, highly contagious disease that, although considered a childhood illness, is often seen in the United States in adolescents and young adults. Symptoms include fever, sneezing, coughing, conjunctivitis, small white spots on the inside of the cheek (Koplik spots), and a red, blotchy rash beginning several days after the respiratory signs. Measles is serious. Around 10% of measles cases require hospital admission. It can lead to pneumonia and encephalitis, and it can kill. Persons who may have been exposed should be informed that anyone under 18 who has not received both immunization doses should receive measles vaccine.

Which of the following components of the epidemiologic triangle contributes most to a female client developing a vaginal infection caused by fungi after successful treatment of her strep throat with antibiotics? a) Agent b) Environment c) Host d) Agent and host

B The antibiotic therapy eliminates a specific pathological agent, but it also may alter the balance of normally occurring organisms in the woman's body, which causes a change in the vaginal environment and allows normally present fungi to proliferate, resulting in a yeast infection.

Which of the following best describes the current goal in relation to communicable diseases? a) To control political borders so diseases cannot spread further b) To exterminate specific infectious agents one by one c) To expand health care facilities to improve infectious disease treatment d) To achieve worldwide immunization to control new cases

B The goal of prevention and control programs is to reduce the prevalence of a disease to a level at which it no longer poses a major public health problem. In some cases, diseases may even be eliminated or eradicated. The goal of elimination is to remove a disease from a large geographical area (e.g., a country or region of the world), such as has been done with polio in the Americas. Eradication is the irreversible termination of all transmission of infection by extermination of the infectious agents worldwide, as has been done with smallpox.

For the nurse to fully understand the threat associated with the release of biological agents and participate in an appropriate response, the Centers for Disease Control and Prevention (CDC) has stated that the biological agents most likely to be employed are those that both have a potential for high mortality and can be easily disseminated to produce: a) immediately severe symptoms b) major panic and social disruption c) multisyndrome effect d) person-to-person transmission

B Understanding that the most likely agents to be used in bioterrorism are easily disseminated, have a potential for high mortality, and are intended to produce major panic and social disruption allows the nurse to fully understand the nature of the threat and plan to participate in an appropriate disaster response to minimize dissemination, activate early treatment, and manage the public's concerns. Disaster-response planning addresses these components.

Family health can be defined as a dynamic, changing, relative state of well-being that includes the biological, psychological, sociological, cultural, and spiritual factors of a family system. This family health approach would best include which of the following underlying principles? Select all that apply. A. Assessment of the individual's health does not determine the overall family system's health. B. Family functioning affects the health of individuals. C. Family system assessment specifically addresses the individual's health. D. The individual's health affects family functioning. E. Simultaneous assessment of individual family members and the family system as a whole is important to family health.

B D E

The advanced practice nurse explains that the client has an upper respiratory infection (URI) and suggests several measures that might make the client more comfortable. Which of the following best describes why the nurse doesn't just prescribe antibiotics as the client repeatedly requests? (Select all that apply.) a) Antibiotics are expensive, whereas the support measures would be almost free of cost b) Viral diseases are not affected by antibiotics c) Clinics cannot afford to continually give antibiotics to anyone who asks for them d) The more antibiotics are prescribed, the more infectious agents develop resistance to such drugs

B, D Antibiotics are not effective against viral diseases, a fact found unacceptable to many clients looking for relief from the misery of a cold or flu. The inappropriate prescribing of antibiotics contributes to the growing problem of infectious agents that have developed resistance to once-powerful antibiotics.

Six students order meals at a local restaurant. Which of the following students are at highest risk for illness? (Select all that apply.) a) The first student asks for a salad with chicken strips and dressing on the side b) The second student asks for a hamburger, very rare c) The third student orders a tuna salad sandwich with extra mayonnaise d) The fourth student orders a breakfast meal with two very soft-poached eggs and toast

B, D Escherichia coli O157:H7 can produce a strong cytotoxin that can cause a potentially fatal hemorrhagic colitis. Hamburger is often involved in outbreaks, since the grinding process exposes pathogens on the surface of the whole meat to the interior of the ground meat, effectively mixing the once-exterior bacteria thoroughly throughout the hamburger so that searing the surface no longer suffices to kill all bacteria. Also, hamburger is often made of meat ground from several sources. The best protection against foodborne pathogens is to thoroughly cook food before eating it. Salmonella is also transmitted by undercooked foods such as eggs, poultry, dairy products, and seafood. Consequently, students eating very rare hamburger and undercooked eggs are at high risk.

2. A nurse is caring for Mexican migrant farmworkers. Which of the following conditions are of greatest importance for the nurse to assess? (Select all that apply.) a. Cholera b. Hepatitis c. High blood level of lead d. Malaria

BC

Community and public health nurses (PHNs) practicing in rural locales consistently note which of the following characteristics of their practice environments?

Broad scope of practice, independence and autonomy, and opportunity for community involvement

2. Which of the following best describes a health professional shortage area (HPSA)? a. An area with inadequate health care facilities for residents b. An isolated area of underserved populations within an urban region c. A region with insufficient numbers of health care providers d. A rural region of the United States with a population density of less than 10,000

C

5. A migrant farmworker presents to the clinic reporting an acute onset of severe abdominal pain, nausea, vomiting, diarrhea, and headache with difficulty concentrating. Which of the following conditions would cause such symptoms? a. Appendicitis b. Bacterial gastroenteritis c. Pesticide poisoning d. Viral illness

C

7. Which of the following is the most accurate description of a migrant farmworker? a. A person who does farm work as the primary means of employment, although other work may be done when the seasonal work ends b. A person who immigrates to the United States to "follow the crops" in performing seasonal farm work c. A person who moves from place to place to earn money performing seasonal agricultural work d. A person who specializes in the development of rural land for the purpose of farming

C

9. A migrant farmworker brings his daughter into the clinic with severe heat stroke from being out in the sun. The nurse explains the danger signs and stresses staying cool and drinking lots of water. The man seems to know this already. Which of the following best describes the most likely reason that this happened? a. It was a rare occurrence, which probably won't be repeated. b. The daughter either disobeyed her father and went out to play in the sun or just did not realize how hot she was. c. Children may work on small farms because the family may need the additional income. d. The parents were busy working and didn't realize the child was outside so long.

C

A nurse asks a family member, "What has changed between you and your spouse since your child's head injury?" Which of the following focuses of the family is the nurse assessing? a. The context b. The client c. A system d. A component of society

C

A nurse focuses on the care of the individual while viewing the client's family as a background resource or possible stressor. Which of the following conceptualizations of family does this nurse's view represent? a. Client b. Component of society c. Context d. System

C

A nurse has just met a family and is being doing their family assessment. Which of the following actions should the nurse take before engaging in self-disclosure? a. Confirm the reason for the assessment. b. Demonstrate culture awareness. c. Take time to build trust. d. Understand the family dynamics.

C

A nurse organizes care for a family by focusing on the common tasks of family life and considering a longitudinal view of the family life cycle. Which theory is being applied? a. Family systems b. Structural-functional c. Family developmental d. Interactionist

C

In taking a family history, the nurse in community health finds that this is the second marriage for the previously divorced parents and that the male partner is the stepparent to the oldest child. For which of the following aspects of the family assessment is data being gathered? a. Dynamics b. Function c. Structure d. System

C

The current-day definition of family refers to two or more individuals who depend on one another for emotional, physical, and/or financial support. Which of the following is the most important principle to support this broader definition? A. Families are defined by genetic ties. B. Family names are needed to confer status. C. Members of a family are self-defined. D. Traditional family functions have been redefined.

C

Immunity to disease through vaccination is known as: a) natural immunity b) resistance c) acquired immunity d) herd immunity

C Acquired immunity is the resistance acquired by a host as a result of previous natural exposure to an infectious agent. Having measles once protects against future infection. Acquired immunity may be induced by active or passive immunization. Natural immunity refers to species-determined, innate resistance to an infectious agent. Resistance is the ability of the host to withstand infection, and it may involve natural or acquired immunity. Herd immunity refers to the immunity of a group or community.

Which of the following is the most common vector-borne disease in the United States? a) Babesiosis b) Ehrlichiosis c) Lyme disease d) Rocky Mountain spotted fever

C All four are diseases borne by ticks as the vectors. Lyme disease became a nationally notifiable disease in 1991 and is now the most common vector-borne disease in the United States.

A man loudly protests his increased property tax bill right after the public health department has made a plea for more funds. "Why," he asks, "should my tax dollars be used to pay for their children to be immunized?" Which of the following would be the best response by the nurse? a) "Immunizations are required by law, and if their parents can't afford it, you and I will have to pay for it." b) "It's just the right thing to do." c) "Only by making sure most kids are immunized can we stop epidemics that might hurt all of us." d) "We're a religious God-fearing community, and we take care of each other."

C Herd immunity is the resistance of a group of people to invasion and spread of an infectious agent because a high proportion of individual members of a group are resistant to the infection. Higher immunization coverage will lead to greater herd immunity, which in turn will block the further spread of the disease

An instructor is reviewing Salmonella infections with her class. Which of the following comments indicates that the student needs further review on how Salmonella is spread? a) "Certain pets and farm animals may be Salmonella carriers." b) "It is possible to transmit Salmonella by person-to-person contact." c) "Salmonella may be spread by spores that form once contaminated blood is exposed to the air." d) "Salmonella outbreaks are usually due to contaminated meat, poultry, and eggs."

C Meat, poultry, and eggs are the foods most often associated with salmonellosis outbreaks. Animals are the common reservoir for the various Salmonella serotypes, although infected humans may also fill this role. Animals are more likely to be chronic carriers. Reptiles such as iguanas have been implicated as Salmonella carriers, along with pet turtles, poultry, cattle, swine, rodents, dogs, and cats. Person-to-person transmission is an important consideration in daycare and institutional settings. Anthrax (not Salmonella) forms spores when infected blood is exposed to air.

A community is experiencing an epidemic of the measles. The nurse is trying to determine if this problem is happening in other communities as well. Which of the following resources should the nurse use to answer this question? a) Centers for Disease Control and Prevention Weekly Report b) Communicable Diseases Weekly Report c) Morbidity and Mortality Weekly Report d) Weekly National Report of Communicable Diseases

C Requirements for disease reporting in the United States are mandated by state rather than federal law. The list of reportable diseases varies by state. State health departments, on a voluntary basis, report cases of selected diseases to the Centers for Disease Control and Prevention (CDC) in Atlanta, Georgia. The National Notifiable Diseases Surveillance System (NNDSS) data are collated and published weekly in the Morbidity and Mortality Weekly Report (MMWR).

Which of the following data would most likely be collected in a syndromic surveillance system? a) Incidence of bioterrorism attacks b) Number of air travelers c) Incidence of school absenteeism d) Number of influenza vaccines administered

C Syndromic surveillance systems use existing health data in real time to provide immediate analysis and feedback to those charged with investigation and follow-up of potential outbreaks. These systems incorporate factors such as the previously mentioned temporal and geographic clustering and unusual age distributions with groups of disease symptoms or syndromes (e.g., flaccid paralysis, respiratory signs, skin rashes, gastrointestinal symptoms) with the goal of detecting early signs of diseases that could result from a bioterrorism-related attack. Syndromic surveillance systems may include tracking emergency department visits sorted by syndrome symptoms as well as other indicators of illness including school absenteeism and sales of selected over-the-counter medications. In recent years, the tracking of cold medicines used to make crystal methamphetamine has received considerable attention.

An example of secondary prevention for infectious disease prevention is: a) malaria chemoprophylaxis b) Pneumocystis carinii pneumonia (PCP) chemoprophylaxis for people with AIDS c) quarantine d) restaurant inspections

C The goal of secondary prevention is to prevent the spread of disease once it occurs. Activities center on rapid identification of potential contacts to a reported case.Primary prevention interventions prevent the occurrence of disease, and tertiary prevention interventions reduce the complications and disabilities through treatment and rehabilitation.

Which of the following public health actions has been particularly instrumental in reducing childhood infectious diseases in the United States? a) Answering parents' questions about the safety and importance of vaccines today b) Educational campaigns to all health care providers about the importance of immunizations whenever a child is seen c) "No shots, no school" legislation, which legally requires children be immunized before school d) Offering all immunizations to all children free of any charge

C Vaccines are one of the most effective methods of preventing and controlling communicable diseases. Hopefully, all nurses answer questions, remind colleagues to think about immunizations whenever a child is seen, and encourage continuing free or low-cost immunization clinics. One of the most effective programs has been the "no shots, no school" legislation, which has resulted in the immunization of most children by the time they enter school.

In comparison with traditional norms, which family functions have become increasingly important in modern American society? (Select all that apply.) a. Conferring appropriate social status b. Educating the younger members c. Ensuring physical and mental health d. Fostering interpersonal relationships and support

C D

Which of the following symptoms suggests smallpox as opposed to the more common and much less dangerous chickenpox? (Select all that apply.) a) Child appears only mildly ill until late stages in smallpox b) Lesions appear in various stages in the same area of the body rather than all at once c) Rash lesions are most abundant on the face and extremities, not on the trunk d) Rash occurs 2 to 4 days after sudden onset of fever rather than with the fever

C, D Symptoms of smallpox include rash lesions on face and extremities, rash that occurs 2 to 4 days after onset of fever, and vesicles that are deep seated.

13. A nurse is implementing a tertiary prevention strategy related to pesticide exposure. Which of the following activities would the nurse complete? a. Observe farmworkers for evidence of unsafe handling of pesticides. b. Provide teaching on how to handle pesticides to avoid or decrease exposure. c. Teach farmworkers how to recognize signs and symptoms of pesticide poisoning. d. Treat a client who has pesticide exposure to prevent complications.

D

8. An employer provides a migrant farm family the day off to visit the health clinic in a nearby community and tells them to take all of the time they need. However, the family arrives at the clinic appearing very stressed. In addition to the health issue, which of the following would most likely be a fear experienced by the family? a. Their personal belongings may be stolen while they are at the clinic. b. Immigration officials will send them back to their home country. c. The clinic personnel will look down on them and be biased against them. d. They weren't getting paid for that day, and continued employment is never certain.

D

A nurse is conducting a family assessment. Which of the following behaviors would the nurse recognize as suggestive of a family with problems? a. Before eating, the family prayed, expressing gratitude for their blessings. b. During family play, jokes and laughter were heard. c. Each person had a private room with a door for alone time. d. Most of the conversation was between the father and the eldest daughter.

D

A nurse is using the provisions of the Family Medical Leave legislation. Which of the following actions is the nurse most likely to take? a. Resigning from employment, but retaining health insurance b. Sharing family information with colleagues c. Providing Medicaid to a family who cannot afford health insurance d. Taking a defined time off of work for family events without fear of job loss

D

The family systems theory encourages nurses to view both the individual clients as participating members of a whole family. What is the major weakness of the systems framework? A. Views families from both a subsystem and a suprasystem approach B. Defines the direction of interactions C. Views the family as an agent of change D. Focuses on the interaction of the family with other systems

D

The following people enter the health clinic together: an unmarried man and his year-old son, an unmarried woman with a year-old daughter, and the man's married brother, who is separated from his wife. During the assessment it is determined that both men work and contribute to the household, where all of them live. Which of the following best describes the "family"? a. The group consists of three families: the man and his son, the woman and her daughter, and the brother, who is married even though he and his wife are separated. b. There are two families involved: first, the unmarried man and woman and their two children, and second, the brother, who is married even though he and his wife are separated. c. There is no family here, only three adults sharing resources between themselves and two biologically related children. d. The family includes whoever the adults state are family members.

D

The nurse in community health needs to conduct a family assessment within a commune but is uncertain how to proceed because family lines appear blurred. The best way to determine the family of a mother and her child is to ask the woman which of the following questions? a. "How many children do you have, and who is the father of each?" b. "Is there a register of families who are members of this commune?" c. "Tell me about your significant other." d. "Who are the members of your and your child's family?"

D

Which of the following social science theories is used by public health nursing to describe how environments and systems outside of the family influence the development of a child over time? A. Life cycle theory B. Family developmental theory C. Family systems theory D. Bioecological systems theory

D

Which of the following terms refers to government actions that have a direct or indirect effect on families? a. Family funding b. Family legislation c. Family planning d. Family policy

D

Which of the following theories views the family as a whole with boundaries that are affected by the environment? a. Family developmental theory b. Structural-functional theory c. Family role theory d. Family systems theory

D

A student engages in unprotected sex under the influence of alcohol. The student decides to have an HIV test completed the next day. Which of the following results will most likely occur? a) The results will probably be negative for HIV b) The results will probably be positive for HIV c) The probability of disease is so low there is no reason to be tested d) The test results won't be reliable so soon after exposure

D It may take up to 6 months after exposure to the HIV virus before an HIV antibody test can test positive, although most infected people will test positive within 3 months. A negative test, therefore, is not a reliable indicator of infection status if exposure is very recent. The incubation period or the time interval between invasion by an infectious agent and the first appearance of signs and symptoms of the disease may be between 10 and 15 years for AIDS.

There is great concern in the nurse's community over three local cases of West Nile virus. Which of the following actions should the nurse take to get the community involved in addressing this problem? a) Ask the state department of health for assistance b) Demand that everyone over age 65 become immunized immediately c) Encourage immunization of all children under 12 d) Have an educational campaign to remove any containers of standing water

D Periodic outbreaks of West Nile virus appear to result from a complex interaction of multiple factors, including weather—especially hot, dry summers followed by rain, which influences mosquito breeding sites and population growth. Removing standing water will remove mosquito breeding sites.

Which of the following infectious disease interventions best represents the use of secondary prevention? a) Clients with HIV infection are encouraged to use condoms to protect sexual partners b) Clients with HIV infection are given medications to improve immunological response c) Health care workers are encouraged to receive annual vaccinations for influenza d) Health care workers are required to have a tuberculosis skin test or chest x-ray

D Tuberculosis screening for health care workers is an example of secondary prevention. TB skin tests and chest x-rays are methods of tuberculosis screening. Encouraging clients with HIV to use condoms is an example of primary prevention, because the goal is to prevent exposure to the partner. Encouraging annual influenza vaccinations is also an example of primary prevention. Giving clients with HIV medications is an example of tertiary prevention, because the goal is to reduce complications in those already having the infection.

At a town meeting with public health officials to discuss a communicable disease outbreak, a nurse is asked to explain what is meant by the phrase "a virulent organism." The nurse explains that this means the organism causing the disease is able to do which of the following? a) Bypass normal immunological response mechanisms b) Invade major organ systems c) Produce toxins and poisons that weaken the body d) Produce very severe physical reactions

D Virulence is the ability to produce a severe pathological reaction.

Depression among rural residents appears to be more persistent and endemic. Which of the following factors may contribute to this level of depression? (Select all that apply.)

a. Delays in seeking mental health services b. High rate of poverty c. Gaps in the continuum of mental health services e. Tolerance for destructive coping mechanisms

A community-oriented nurse newly assigned to a rural community learns that the characteristics of rural and small town life include:

nformal social and professional relationships, acquaintance of residents with most other members of the community, work of many residents in high-risk occupations, and often a lack of openness to newcomers.

Nurses practicing in rural communities often observe that protecting client confidentiality is a unique challenge because:

nurses in rural areas are well known to their service populations and are often approached by their clients in social and other settings with requests for counsel and advice.

When determining whether a geographic area is rural or urban, the nurse should recognize that:

rural and urban areas by their nature occur on a continuum.


संबंधित स्टडी सेट्स

SHSU - POLS3393 - Social Policy International Disaster Management

View Set

The Integumentary System (Chapter 5)

View Set

Chapter 2: Commercial Sponsorship

View Set

(pharm) Ch 48: Drug Therapy for Parkinson's Disease, Urinary Spasticity, and Disorders Requiring Anticholinergic Therapy

View Set

Biomaterial Test 2 Practice Problem

View Set

Chapter 16 Environmental Science

View Set